Физика Электродвижущая сила. Закон Ома для полной цепи
Материалы к уроку
Конспект урока
Соединим проводником два металлических шарика, несущих заряды противоположных знаков. Под влиянием электрического поля этих зарядов в проводнике возникает электрический ток. Но этот ток будет кратковременным. Заряды быстро нейтрализуются, потенциалы шариков станут одинаковыми, и электрическое поле исчезнет. Для того чтобы ток был постоянным, надо поддерживать постоянное напряжение между шариками. Для этого необходимо устройство (источник тока), которое перемещало бы заряды от одного шарика к другому в направлении противоположном направлению сил, действующих на эти заряды со стороны электрического поля шариков.
В таком устройстве на заряды, кроме электрических сил должны действовать силы и неэлектрического происхождения. Одно лишь электрическое поле заряженных частиц (кулоновское поле) не способно поддерживать постоянный ток в цепи. Любые силы, действующие на электрически заряженные частицы, за исключением сил электростатического происхождения (т.е. кулоновских), называют сторонними силами. Сторонние силы приводят в движение заряженные частицы внутри всех источников тока: в генераторах на электростанциях, в гальванических элементах, аккумуляторах.
Природа сторонних сил может быть разнообразной. В генераторах электростанций сторонняя сила – эта сила, действующая со стороны магнитного поля на электроны в движущемся проводнике.
В гальваническом элементе, например, в элементе Вольта, действуют химические силы.
Первая электрическая батарея появилась в 1799 году. Её изобрел итальянский физик Алессандро ВОльта (1745 — 1827) — итальянский физик, химик и физиолог, изобретатель источника постоянного электрического тока. Его первый источник тока – «вольтов столб» был построен в точном соответствии с его теорией «металлического» электричества. Вольта положил друг на друга попеременно несколько десятков небольших цинковых и серебряных кружочков, проложив меж ними бумагу, смоченную подсоленной водой.
До конца XVIII века все технические источники тока были основаны на электризации трением. Наиболее эффективным из этих источников стала электрофорная машина (диски машины приводятся во вращение в противоположных направлениях. В результате трения щеток о диски на кондукторах машины накапливаются заряды противоположного знака) Сторонние силы возникают в результате трения.
При освещении некоторых веществ светом в них появляется ток, световая энергия превращается в электрическую.
В данном приборе заряды разделяются под действием света. Фотоэлементы применяются в солнечных батареях, световых датчиках, калькуляторах, видеокамерах.
Сторонние силы возникают под действием света.
Если две проволоки из разных металлов спаять с одного края, а затем нагреть место спая, то в них возникнет ток — заряды при нагревании спая разделяются. Термоэлементы применяются в термодатчиках и на геотермальных электростанциях в качестве датчика температуры.
Сторонние силы возникают под действием тепла.
Действие сторонних сил характеризуется важной физической величиной электродвижущей силой (сокращенно ЭДС). Электродвижущая сила в замкнутом контуре представляет собой отношение работы сторонних сил при перемещении заряда вдоль контура к заряду. Электродвижущую силу выражают в вольтах. Это удельная работа сторонних сил не во всем контуре, а только на данном участке, т.е. работа по перемещению единичного заряда.
Электродвижущая сила определяет силу тока в замкнутой электрической цепи с известным сопротивлением. Рассмотрим простейшую полную (замкнутую) цепь, состоящую из источника тока (гальванического элемента, аккумулятора или генератора) и резистора сопротивлением (эр большое) R .
Пример. ЭДС батареи 6,0 В, ее внутреннее сопротивление 0,5 Ом, сопротивление внешней цепи 11,5 Ом. Найдите силу тока в цепи, напряжение на зажимах батареи и падение напряжения внутри батареи.
Пусть R (эр большое) — сопротивление внешнего участка цепи, r (эр малое) — внутреннее сопротивление батареи.
Тогда по закону Ома для замкнутой цепи (формула), где ε(эпсилон) — ЭДС батареи, I (и)- сила тока в цепи. Так как сила тока I одинакова как для внешнего, так и для внутреннего участков цепи, то напряжение на зажимах батареи, т. е. на внешнем участке цепи с сопротивлением R , по закону Ома для этого однородного участка есть формула.
Подставляем значения и проводим расчеты.
Получаем ответ: сила тока в цепи равна 0,5 А; напряжение на зажимах батареи 5,75 В; падение напряжения на внутреннем сопротивлении 0,25В.
Внутреннее сопротивление источника тока не оказывает заметного влияния на силу тока, если оно мало по сравнению с сопротивлением внешней части цепи (R>>r). При этом напряжение на зажимах источника приблизительно равно ЭДС.
При коротком замыкании, сила тока в цепи определяется именно внутренним сопротивлением источника и при электродвижущей силе в несколько вольт может оказаться очень большим, если r мало. Провода могут расплавиться, а сам источник выйти из строя.
Для защиты от короткого замыкания принимают специальные меры.
Устанавливают токоограничивающие электрические реакторы, применяют распараллеливание электрических цепей, т.е. отключение секционных и шиносоединительных выключателей, используют понижающие трансформаторы с расщепленной обмоткой низкого напряжения; используют отключающее оборудование – быстродействующие коммутационные аппараты с функцией ограничения тока короткого замыкания – плавкие предохранители и автоматические выключатели; применяют устройства релейной защиты для отключения поврежденных участков цепи.
Знак ЭДС определяется произвольно по выбранному направлению обхода контура. Если при обходе переходим от отрицательного полюса к положительному, то ЭДС будет положительной.
Как вы думаете, какое напряжение может представлять опасность для жизни человека?
Опасная для жизни человека сила тока равна 0,05 ампер.
Остались вопросы по теме? Наши репетиторы готовы помочь!
Подготовим к ЕГЭ, ОГЭ и другим экзаменам
Найдём слабые места по предмету и разберём ошибки
Повысим успеваемость по школьным предметам
Поможем подготовиться к поступлению в любой ВУЗ
Выбрать репетитора
Внутреннее сопротивление источника тока. Сопротивление
Лабораторная работа
«Измерение ЭДС и внутреннего сопротивления источника тока»
Дисциплина Физика
Преподаватель Виноградов А.Б.
Нижний Новгород
2014 г.
Цель работы: сформировать умение определения ЭДС и внутреннего сопротивления источника тока с помощью амперметра и вольтметра.
Оборудование: выпрямитель ВУ-4М, амперметр, вольтметр, соединительные провода, элементы планшета №1: ключ, резистор R 1 .
Теоретическое содержание работы .
Внутреннее сопротивление источника тока.
При прохождении тока по замкнутой цепи, электрически заряженные частицы перемещаются не только внутри проводников, соединяющих полюса источника тока, но и внутри самого источника тока. Поэтому в замкнутой электрической цепи различают внешний и внутренний участки цепи. Внешний участок цепи составляет вся та совокупность проводников, которая подсоединяется к полюсам источника тока. Внутренний участок цепи — это сам источник тока. Источник тока, как и любой другой проводник, обладает сопротивлением. Таким образом, в электрической цепи, состоящей из источника тока и проводников с электрическим сопротивлением R , электрический ток совершает работу не только на внешнем, но и на внутреннем участке цепи. Например, при подключении лампы накаливания к гальванической батарее карманного фонаря электрическим током нагреваются не только спираль лампы и подводящие провода, но и сама батарея. Электрическое сопротивление источника тока называется внутренним сопротивлением. В электромагнитном генераторе внутренним сопротивлением является электрическое сопротивление провода обмотки генератора. На внутреннем участке электрической цепи выделяется количество теплоты, равное
где r — внутреннее сопротивление источника тока.
Полное количество теплоты, выделяющееся при протекании постоянного тока в замкнутой цепи, внешний и внутренний участки которой имеют сопротивления, соответственно равные R и r , равно
. (2)
Всякую замкнутую цепь можно представить как два последовательно соединенных резистора с эквивалентными сопротивлениями R и r . Поэтому сопротивление полной цепи равно сумме внешнего и внутреннего сопротивлений:
. Поскольку при последовательном соединении сила тока на всех участках цепи одинакова, то через внешний и внутренний участок цепи проходит одинаковый по величине ток. Тогда по закону Ома для участка цепи падение напряжений на ее внешнем и внутреннем участках будут соответственно равны:
и
(3)
Электродвижущая сила.
Полная работа сил электростатического поля при движении зарядов по замкнутой цепи постоянного тока равна нулю. Следовательно, вся работа электрического тока в замкнутой электрической цепи оказывается совершенной за счет действия сторонних сил, вызывающих разделение зарядов внутри источника и поддерживающих постоянное напряжение на выходе источника тока. Отношение работы
, совершаемой сторонними силами по перемещению заряда q вдоль цепи, к значению этого заряда называется электродвижущей силой источника (ЭДС) :
, (4)
где
— переносимый заряд.
ЭДС выражается в тех же единицах, что и напряжение или разность потенциалов, т. е. в вольтах:
.
Закон Ома для полной цепи.
Если в результате прохождения постоянного тока в замкнутой электрической цепи происходит только нагревание проводников, то по закону сохранения энергии полная работа электрического тока в замкнутой цепи, равная работе сторонних сил источника тока, равна количеству теплоты, выделившейся на внешнем и внутреннем участках цепи:
. (5)
Из выражений (2), (4) и (5) получаем:
. (6)
Так как
, то
, (7)
или
. (8)
Сила тока в электрической цепи прямо пропорциональна электродвижущей силе источника тока и обратно пропорциональна сумме электрических сопротивлений внешнего и внутреннего участков цепи. Выражение (8) называется законом Ома для полной цепи.
Таким образом, с точки зрения физики Закон Ома выражает закон сохранения энергии для замкнутой цепи постоянного тока.
Порядок выполнения работы .
Подготовка к выполнению работы.
Перед вами на столах находится минилаборатория по электродинамике. Её вид представлен в л. р. № 9 на рисунке 2.
Слева находятся миллиамперметр, выпрямитель ВУ-4М, вольтметр, амперметр. Справа закреплен планшет № 1 (см. рис. 3 в л. р. № 9). В задней секции корпуса размещаются соединительные провода цветные: красный провод используют для подключения ВУ-4М к гнезду «+» планшета; белый провод — для подключения ВУ-4М к гнезду «-»; желтые провода — для подключения к элементам планшета измерительных приборов; синие — для соединения между собой элементов планшета. Секция закрыта откидной площадкой. В рабочем положении площадка располагается горизонтально и используется в качестве рабочей поверхности при сборке экспериментальных установок в опытах.
2. Ход работы.
В ходе работы вы освоите метод измерения основных характеристик источника тока, используя закон Ома для полной цепи, который связывает силу тока I в цепи, ЭДС источника тока , его внутреннее сопротивление r и сопротивление внешней цепи R соотношением:
. (9)
1 способ.
Схема экспериментальной установки показана на рисунке 1.
Рис.1.
Внимательно изучите её. При разомкнутом ключе В источник замкнут на вольтметр, сопротивление которого много больше внутреннего сопротивления источника (r R ). В этом случае ток в цепи настолько мал, что можно пренебречь значением падения напряжения на внутреннем сопротивлении источника
, и ЭДС источника с пренебрежимо малой погрешностью равна напряжения на его зажимах , которое измеряется вольтметром, т.е.
. (10)
Таким образом, ЭДС источника определяется по показаниям вольтметра при разомкнутом ключе В.
Если ключ В замкнуть, вольтметр покажет падение напряжения на резисторе R :
. (11)
Тогда на основании равенств (9), (10) и (11) можно утверждать, что
(12)
Из формулы (12) видно, что для определения внутреннего сопротивления источника тока необходимо, кроме его ЭДС, знать силу тока в цепи и напряжение на резисторе R при замкнутом ключе.
Силу тока в цепи можно измерить при помощи амперметра. Проволочный резистор изготовлен из нихромовой проволоки и имеет сопротивление 5 Ом.
Соберите цепь по схеме, показанной на рисунке 3.
После того, как цепь будет собрана, необходимо поднять руку, позвать учителя, чтобы он проверил правильность сборки электрической цепи. И если цепь собрана правильно, то приступайте к выполнению работы.
При разомкнутом ключе В снимите показания вольтметра и занесите значение напряжения в таблицу 1. Затем замкните ключ В и опять снимите показания вольтметра, но уже и показания амперметра. Занесите значение напряжения и силы тока в таблицу 1.
Вычислите внутреннее сопротивление источника тока.
Таблица1.
, В
, В
I , А
, В
r , Ом
2 способ.
Сначала соберите экспериментальную установку, изображенную на рисунке 2.
Рис. 2.
Измерьте силу тока в цепи при помощи амперметра, результат запишите в тетрадь. Сопротивление резистора =5 Ом. Все данные заносятся в таблицу 2. , Ом
Контрольные вопросы :
Внешний и внутренний участки цепи.
Какое сопротивление называются внутренним? Обозначение.
Чему равно полное сопротивление?
Дайте определение электродвижущей силы (ЭДС). Обозначение. Единицы измерения.
Сформулируйте закон Ома для полной цепи.
Если бы мы не знали значения сопротивлений проволочных резисторов, то можно ли было бы использовать второй способ и что для этого надо сделать (может нужно, например, включить в цепь какой-нибудь прибор)?
Уметь собирать электрические цепи, используемые в работе.
Литература
Кабардин О. Ф.. Справ. Материалы: Учеб. Пособие для учащихся.-3-е изд.-М.:Просвещение,1991.-с.:150-151.
Справочник школьника. Физика/ Сост. Т. Фещенко, В. Вожегова.–М.: Филологическое об-щество «СЛОВО», ООО «Фирма» «Издательство АСТ», Центр гуманитарных наук при ф-те журна-листики МГУ им. М. В. Ломоносова, 1998. — с.: 124,500-501.
Самойленко П. И.. Физика (для нетехнических специальностей): Учебн. для общеобразоват. учреждений сред. Проф. Образования/ П. И.Самойленко, А. В. Сергеев.-2-е изд., стер.-М.: Издательский центр «Академия», 2003-с.: 181-182.
Цель работы: изучить метод измерения ЭДС и внутреннего сопротивления источника тока с помощью амперметра и вольтметра.
Оборудование: металлический планшет, источник тока, амперметр, вольтметр, резистор, ключ, зажимы, соединительные провода.
Для измерения ЭДС и внутреннего сопротивления источника тока собирают электрическую цепь, схема которой показана на рисунке 1.
К источнику тока подключают амперметр, сопротивление и ключ, соединенные последовательно. Кроме того, непосредственно к выходным гнездам источника подключают еще и вольтметр.
ЭДС измеряют по показанию вольтметра при разомкнутом ключе. Этот прием определения ЭДС основан на следствии из закона Ома для полной цепи, согласно которому при бесконечно большом сопротивлении внешней цепи напряжение на зажимах источника равно его ЭДС. (См. параграф «Закон Ома для полной цепи» учебника «Физика 10»).
Для определения внутреннего сопротивления источника замыкают ключ К. При этом в цепи можно условно выделить два участка: внешний (тот, который подключен к источнику) и внутренний (тот, который находится внутри источника тока). Поскольку ЭДС источника равна сумме падения напряжений на внутреннем и внешнем участках цепи:
ε = U r +U R , то U r = ε -U R (1)
По закону Ома для участка цепи U r = I· r (2). Подставив равенство (2) в (1) получают:
I · r = ε — U r , откуда r = (ε — U R )/ J
Следовательно, чтобы узнать внутреннее сопротивление источника тока, необходимо предварительно определить его ЭДС, затем замкнуть ключ и измерить падение напряжения на внешнем сопротивлении, а также силу тока в нем.
Ход работы
1. Подготовьте таблицу для записи результатов измерений и вычислений:
ε ,в | U r , B | i,a | r , Ом |
Начертите в тетради схему для измерения ЭДС и внутреннего сопротивления источника.
После проверки схемы соберите электрическую цепь. Ключ разомкните.
Измерьте величину ЭДС источника.
Замкните ключ и определите показания амперметра и вольтметра.
Вычислите внутреннее сопротивление источника.
Определение эдс и внутреннего сопротивления источника тока графическим методом
Цель работы: изучить измерения ЭДС, внутреннего сопротивления и тока короткого замыкания источника тока, основанный на анализе графика зависимости напряжения на выходе источника от силы тока в цепи.
Оборудование: гальванический элемент, амперметр, вольтметр, резистор R 1 , переменный резистор, ключ, зажимы, металлический планшет, соединительные провода.
Из закона Ома для полной цепи следует, что напряжение на выходе источника тока зависит прямо пропорционально от силы тока в цепи:
так как I =E/(R+r), то IR + Ir = Е, но IR = U, откуда U + Ir = Е или U = Е – Ir (1).
Если построить график зависимости U от I, то по его точкам пересечения с осями координат можно определить Е, I К.З. — силу тока короткого замыкания (ток, который потечет в цепи источника, когда внешнее сопротивление R станет равным нулю).
ЭДС определяют по точке пересечения графика с осью напряжений. Эта точка графика соответствует состоянию цепи, при котором ток в ней отсутствует и, следовательно, U = Е.
Силу тока короткого замыкания определяют по точке пересечения графика с осью токов. В этом случае внешнее сопротивление R = 0 и, следовательно, напряжение на выходе источника U = 0.
Внутреннее сопротивление источника находят по тангенсу угла наклона графика относительно оси токов. (Сравните формулу (1) с математической функцией вида У = АХ +В и вспомните смысл коэффициента при X).
Ход работы
- После проверки схемы преподавателем соберите электрическую цепь. Ползунок переменного резистора установите в положение, при котором сопротивление цепи, подключенной к источнику тока, будет максимальным.
Для записи результатов измерений подготовьте таблицу:
Определите значение силы тока в цепи и напряжение на зажимах источника при максимальной величине сопротивления переменного резистора. Данные измерений занесите в таблицу.
Повторите несколько раз измерения силы тока и напряжения, уменьшая всякий раз величину переменного сопротивления так, чтобы напряжение на зажимах источника уменьшалось на 0,1В. Измерения прекратите, когда сила тока в цепи достигнет значения в 1А.
Нанесите полученные в эксперименте точки на график. Напряжение откладывайте по вертикальной оси, а силу тока — по горизонтальной. Проведите по точкам прямую линию.
Продолжите график до пересечения с осями координат и определите величины Е и, I К.З.
Измерьте ЭДС источника, подключив вольтметр к его выводам при разомкнутой внешней цепи. Сопоставьте значения ЭДС, полученные двумя способами, и укажите причину возможного расхождения результатов.
Определите внутреннее сопротивление источника тока. Для этого вычислите тангенс угла наклона построенного графика к оси токов. Так как тангенс угла в прямоугольном треугольнике равен отношению противолежащего катета к прилежащему, то практически это можно сделать, найдя отношение Е / I К.З
Мы пришли к выводу, что для поддержания постоянного тока в замкнутой цепи, в нее необходимо включить источник тока. Подчеркнем, что задача источника заключается не в том, чтобы поставлять заряды в электрическую цепь (в проводниках этих зарядов достаточно), а в том, чтобы заставлять их двигаться, совершать работу по перемещению зарядов против сил электрического поля. Основной характеристики источника является электродвижущая сила 1 (ЭДС) − работа, совершаемая сторонними силами по перемещению единичного положительного заряда
Поэтому большинству людей нужны ассоциации или критическая масса в планетарном поле, чтобы получать сигналы энергии и воспоминания о сознании и иметь возможность правильно воспринимать сигналы. Трехмерная система управления не учитывает симптомы вознесения, опыт, связанный с сознанием, или многие радикальные изменения, которые происходят у людей с этой Земли. Заземление — это форма заземления на Земле и относится к прямому контакту тела с элементами Земли. Это может быть полезно для многих людей, которые испытывают недостаток заземления и плотского дискомфорта во время планетарных изменений.
Единицей измерения ЭДС в системе единиц СИ является Вольт. ЭДС источника равна 1 вольт, если он совершает работу 1 Джоуль при перемещении заряда 1 Кулон
Для обозначения источников тока на электрических схемах используется специальное обозначение (рис. 397).
рис. 397
Электростатическое поле совершает положительную работу по перемещению положительного заряда в направлении уменьшения потенциала поля. Источник тока проводит разделение электрических зарядов − на одном полюсе накапливаются положительные заряды, на другом отрицательный. Напряженность электрического поля в источнике направлена от положительного полюса к отрицательному, поэтому работа электрического поля по перемещению положительного заряда будет положительной при его движения от «плюса» к «минусу». Работа сторонних сил, наоборот, положительна в том случае, если положительные заряды перемещаются от отрицательного полюса к положительному, то есть от «минуса» к «плюсу».
В этом принципиальное отличие понятий разности потенциалов и ЭДС, о котором всегда необходимо помнить.
Таким образом, электродвижущую силу источника можно считать алгебраической величиной, знак которой («плюс» или «минус») зависит от направления тока. В схеме, показанной на рис. 398,
рис. 398
вне источника (во внешней цепи) ток течет 2 от «плюса» источника к «минусу», в внутри источника от «минуса» к «плюсу». В этом случае, как сторонние силы источника, так и электростатические силы во внешней цепи совершают положительную работу.
Если на некотором участке электрической цепи помимо электростатических действуют и сторонние силы, то над перемещением зарядов «работают» как электростатические, так и сторонние силы. Суммарная работа электростатических и сторонних сил по перемещению единичного положительного заряда называется электрическим напряжением на участке цепи
В том случае, когда сторонние силы отсутствуют, электрическое напряжение совпадает с разностью потенциалов электрического поля.
Поясним определение напряжения и знака ЭДС на простом примере. Пусть на участке цепи, по которому протекает электрический ток, имеются источник сторонних сил и резистор (рис. 399).
рис. 399
Для определенности будем считать, что φ o > φ 1 , то есть электрический ток направлен от точки 0 к точке 1 . При подключении источника, как показано на рис. 399 а, Сторонние силы источника совершают положительную работу, поэтому соотношение (2) в этом случае может быть записано в виде
При обратном включении источника (рис. 399 б) внутри него заряды движутся против сторонних сил, поэтому работа последних отрицательна. Фактически силы внешнего электрического поля преодолевают сторонние силы. Следовательно, в этом случае рассматриваемое соотношение (2) имеет вид
Для протекания электрического тока по участку цепи, обладающему электрическим сопротивлением, необходимо совершать работу, по преодолению сил сопротивления. Для единичного положительного заряда эта работа, согласно закону Ома, равна произведению IR = U которое, естественно совпадает с напряжением на данном участке.
Заряженные частицы (как электроны, так и ионы) внутри источника движутся в некоторой , поэтому со стороны среду на них также действуют тормозящие силы, которые также необходимо преодолевать. Заряженные частицы преодолевают силы сопротивления благодаря действию сторонних сил (если ток в источнике направлен от «плюса» к «минусу») либо благодаря электростатическим силам (если ток направлен от «минуса» к «плюсу»). Очевидно, что работа по преодолению этих сил не зависит от направления движения, так как силы сопротивления всегда направлены в сторону, противоположную скорости движения частиц. Так как силы сопротивления пропорциональны средней скорости движения частиц, то работа по их преодолению пропорциональна скорости движения, следовательно, силе тока силе. Таким образом, мы можем ввести еще характеристику источника − его внутренне сопротивление r , аналогично обычному электрическому сопротивлению. Работа по преодолению сил сопротивления при перемещении единичного положительного заряда между полюсами источника равна A/q = Ir . Еще раз подчеркнем, эта работа не зависит от направления тока в источнике.
Если у вас нет доступа к природе, и вы хотите создать электрическую схему с полем Земли, вы также можете использовать праймер, который связан с человеческим телом. Электрический потенциал цепи заземления зависит от местоположения, атмосферных условий, времени суток и ночи, а также от влаги, которая расположена на поверхности Земли. Интуитивные эмпаты и звездные саженцы, которые хотят восстановить энергетическую настройку с телом планеты, должны обратить внимание на их естественные чувства, потому что они должны знать, должны ли они быть заземлены или нет.
1 Название этой физической величины неудачно − так электродвижущая сила является работой, а не силой в обычном механическом понимании. Но этот термин настолько устоялся, что изменять его не «в наших силах». К слову, сила тока то же не является механической силой! Не говоря уж о таких понятиях «сила духа», «сила воли», «божественная сила» и т.д.
2 Напомним, за направление движения электрического тока принято направление движения положительных зарядов.
В некоторых случаях из-за неорганических или внешних течений в определенных областях эта практика может оказаться нецелесообразной. Для большинства людей, которые посеяны Землей, на фазе духовной интеграции обоснование будет положительно ощущаться и будет очень полезно для тела, потому что оно будет действовать как нейромодулятор. Нейромодуляция — это процесс, в котором активность нервной системы регулируется путем регулирования физиологических уровней посредством стимуляции нейротрансмиттеров. Таким образом, заземление изменяет плотность отрицательного заряда в области энергии человека и его нервной системы и непосредственно влияет на физиологические процессы, такие как химия мозга.
Лабораторная работа
«Измерение ЭДС и внутреннего сопротивления источника тока»
Дисциплина Физика
Преподаватель Виноградов А.Б.
Нижний Новгород
Цель работы: сформировать умение определения ЭДС и внутреннего сопротивления источника тока с помощью амперметра и вольтметра.
Земля посылает электромагнитные сигналы для поддержки человеческих тел при адаптации к ее вознесению, и этот сигнал позволяет человеческой нервной системе лучше адаптироваться к требованиям, предъявляемым к телу и мозгу во время интенсивных изменений сознания. Когда мы хотим восстановить электрический баланс активности мозга, может быть особенно полезно окружить природу, сосредоточиться на глубоком дыхании и соединиться с Землей или с элементом воды.
Почки — это органы, которые питают энергию. В настоящее время население людей переживает эпидемию заболеваний почек, вызванных неспособностью органов быстро адаптироваться к новым обстоятельствам, плохого признания событий, изменяющих жизнь, сердечных заболеваний, перегрузки токсичными химическими веществами и негативных эмоций. Целью почек является удаление вредных метаболических продуктов, выделяемых мочевым пузырем, и поддержание надлежащей химии крови и давления, поскольку они контролируют все химические вещества, растворенные в кровотоке.
Оборудование: выпрямитель ВУ-4М, амперметр, вольтметр, соединительные провода, элементы планшета №1: ключ, резистор R 1 .
Теоретическое содержание работы .
Внутреннее сопротивление источника тока.
При прохождении тока по замкнутой цепи, электрически заряженные частицы перемещаются не только внутри проводников, соединяющих полюса источника тока, но и внутри самого источника тока. Поэтому в замкнутой электрической цепи различают внешний и внутренний участки цепи. Внешний участок цепи составляет вся та совокупность проводников, которая подсоединяется к полюсам источника тока. Внутренний участок цепи — это сам источник тока. Источник тока, как и любой другой проводник, обладает сопротивлением. Таким образом, в электрической цепи, состоящей из источника тока и проводников с электрическим сопротивлением R , электрический ток совершает работу не только на внешнем, но и на внутреннем участке цепи. Например, при подключении лампы накаливания к гальванической батарее карманного фонаря электрическим током нагреваются не только спираль лампы и подводящие провода, но и сама батарея. Электрическое сопротивление источника тока называется внутренним сопротивлением. В электромагнитном генераторе внутренним сопротивлением является электрическое сопротивление провода обмотки генератора. На внутреннем участке электрической цепи выделяется количество теплоты, равное
Когда почки ослаблены и перегружены, в крови и тканях накапливаются токсичные отходы, а также химические вещества, которые невозможно фильтровать надлежащим образом. Почечная недостаточность увеличивается в Соединенных Штатах на 5% в год, при этом в качестве терапии используют почечный диализ или трансплантацию. Десять процентов населения имеют некоторую форму диабета и неврологического дискомфорта, и это число, по-видимому, неуклонно растет — у взрослых и у детей. Что случилось с нашими почками?
Восточная медицинская философия знает, что почки питают другие органы тела. Они действуют как корни жизни, которые отвечают за защиту организма и распределение энергии во всех органах, репродуктивных функциях и всего организма. Почки — это органы взаимоотношений, поэтому они страдают от проблем с межличностными и сексуальными отношениями, которые могут возникнуть в результате отсутствия поддержки у других или чувства нелюбимой или даже из-за отсутствия физической чувствительности. Эмоции циркулируют в личной энергетической области, и когда она будет выпущена, у вас может возникнуть ощущение течения, благодаря которому вы ощущаете эмоции.
где r — внутреннее сопротивление источника тока.
Полное количество теплоты, выделяющееся при протекании постоянного тока в замкнутой цепи, внешний и внутренний участки которой имеют сопротивления, соответственно равные R и r , равно
Всякую замкнутую цепь можно представить как два последовательно соединенных резистора с эквивалентными сопротивлениями R и r . Поэтому сопротивление полной цепи равно сумме внешнего и внутреннего сопротивлений:
. Поскольку при последовательном соединении сила тока на всех участках цепи одинакова, то через внешний и внутренний участок цепи проходит одинаковый по величине ток. Тогда по закону Ома для участка цепи падение напряжений на ее внешнем и внутреннем участках будут соответственно равны:
Это позволяет вам освобождать эмоциональную боль и страх и избавляет вас от хронических проблем с почками, открывая для себя большее эмоциональное и духовное расширение энергии. Когда это наоборот, когда сердце закрыто от боли и страха, что блокирует эмоции, оно влияет на функцию управления жидкостью через почки и нарушает распределение жизненной энергии, необходимой для заземленного, здорового и сбалансированного ума и тела.
Более того, когда наше сердце исцеляется, внутри горит пламя, которое также питается жизненной энергией, хранящейся в почках. Треугольный соединитель соединяет сердце с каждой почкой, которая работает в светящемся теле, как электрическая цепь. В основании этого треугольника слева и справа находятся почки, а верхняя точка связана с сердцем. Когда сердце исцеляется, пламя в сердце и почках одновременно активирует конфигурацию сердца во внутреннем двойном пламени. Двойное пламя соответствует восстановленному энергетическому балансу между энергией самца и женщины, т.е. структурой света, созданного в комплексе сердца.
и
(3)
Электродвижущая сила.
Полная работа сил электростатического поля при движении зарядов по замкнутой цепи постоянного тока равна нулю. Следовательно, вся работа электрического тока в замкнутой электрической цепи оказывается совершенной за счет действия сторонних сил, вызывающих разделение зарядов внутри источника и поддерживающих постоянное напряжение на выходе источника тока. Отношение работы
, совершаемой сторонними силами по перемещению заряда q вдоль цепи, к значению этого заряда называется электродвижущей силой источника (ЭДС) :
Поэтому, когда два огня зажигаются в сердце, жизненно важная сущность, хранящаяся в почках, помогает переносить чи-пламя по всему физическому телу, чтобы соединиться с духовным пламенем монадического тела. Монада — это большее пламя духа, а физическое тело — меньшее пламя жизненной сущности или жизненной силы. Когда эти два огня зажигаются и объединяются, пламя взрывается от сердца, которое посылает огонь, чтобы поддержать рост сущности жизни, создаваемой почками. В основном, почки помогают построить внутреннее светящееся тело, необходимое для встраивания монадического тела.
, (4)
— переносимый заряд.
ЭДС выражается в тех же единицах, что и напряжение или разность потенциалов, т. е. в вольтах:
.
Закон Ома для полной цепи.
Любые визуальные упражнения, направленные на создание жизненной силы энергии в низших диенах и вызывают энергию для циркуляции у подножия ног, укрепляют способность почек хранить жизненно важную сущность, помогают исправить механизм заземления и выполнять функции физической очистки крови. Существуют некоторые потенцирующие агенты для почек и трав, которые являются общими для восточной медицины и полезны для тонизирования функции почек, особенно если есть проблема с заземлением или центрированием сердечника.
Почечная недостаточность вызывает выработку надпочечников. Надпочечники — это железы, которые производят много гормонов, и хорошо известно, что под давлением они перекачивают кортизол в кровоток, что приводит к тому, что человеческая нервная система переходит в состояние борьбы или полета. Адреналин обычно продуцируется как надпочечниками, так и некоторыми нейронами, которые также могут активироваться эмоциональными реакциями. Каждая эмоциональная реакция имеет поведенческий компонент, компонент вегетативной нервной системы, секрецию железы или гормональный фактор.
Если в результате прохождения постоянного тока в замкнутой электрической цепи происходит только нагревание проводников, то по закону сохранения энергии полная работа электрического тока в замкнутой цепи, равная работе сторонних сил источника тока, равна количеству теплоты, выделившейся на внешнем и внутреннем участках цепи:
Гормональные факторы, связанные со стрессом и эмоциональной болью, включают высвобождение адреналина и реакции надпочечников — в ответ на чувства, основанные на страхе, контролируемые симпатической нервной системой. Основная эмоция, которая выделяет адреналин в кровь, — это страх.
Кроме того, надпочечники играют важную роль в реагировании на борьбу или бегство, увеличивая приток крови к мышцам и сердцу, а затем учащиеся расширяются и уровень сахара в крови увеличивается. Адреналин закачивается в кровоток, когда человек провоцируется на террористические акты или страх, чтобы произвести как можно больше негативной эмоциональной энергии, что может быть основной причиной того, что надпочечники полностью истощены у большинства людей. Когда человек не исправляет это состояние и все еще накачивает адреналин или другие гормоны стресса в кровоток, нервная система замерзает, состояние шока и онемения.
. (5)
Из выражений (2), (4) и (5) получаем:
. (6)
, то
, (7)
В какой-то момент, когда вы испытываете постоянную боль или страх, из-за чрезмерной нагрузки адреналина, тело и нервная система попадают в состояние онемения, которое отключает эмоциональные реакции, закрывая сердце. Надпочечники находятся в верхней части каждой почки, поэтому они непосредственно подвержены истощению почек, что, естественно, приводит к выходу надпочечников. Если мы делаем что-то действительно нездоровое для нашего духа, и наша повседневная работа не соответствует тому, кто мы есть, он также истощает почки, адреналин и жизненную силу.
. (8)
Сила тока в электрической цепи прямо пропорциональна электродвижущей силе источника тока и обратно пропорциональна сумме электрических сопротивлений внешнего и внутреннего участков цепи. Выражение (8) называется законом Ома для полной цепи.
Когда нам приходится сталкиваться с трудными стрессовыми факторами на работе, в отношениях или в других ситуациях, организм может подвергаться глубокому бессознательному эмоциональному стрессу. Мы чувствуем себя беспомощными и расстроены тем, что мы должны просто работать, чтобы выполнить финансовые обязательства или выжить. Наше тело дает нам сообщение из-за чрезмерного истощения, что мы уже не можем жить таким же образом, мы должны вносить изменения, и первое изменение должно состоять в том, чтобы осуществить сознание через смерть эго.
Таким образом, с точки зрения физики Закон Ома выражает закон сохранения энергии для замкнутой цепи постоянного тока.
Порядок выполнения работы .
Подготовка к выполнению работы.
Перед вами на столах находится минилаборатория по электродинамике. Её вид представлен в л. р. № 9 на рисунке 2.
Слева находятся миллиамперметр, выпрямитель ВУ-4М, вольтметр, амперметр. Справа закреплен планшет № 1 (см. рис. 3 в л. р. № 9). В задней секции корпуса размещаются соединительные провода цветные: красный провод используют для подключения ВУ-4М к гнезду «+» планшета; белый провод — для подключения ВУ-4М к гнезду «-»; желтые провода — для подключения к элементам планшета измерительных приборов ; синие — для соединения между собой элементов планшета. Секция закрыта откидной площадкой. В рабочем положении площадка располагается горизонтально и используется в качестве рабочей поверхности при сборке экспериментальных установок в опытах.
Планетарный контроль над человеческими почками Чи. Мы должны стремиться к восстановлению сердечного центра и превращению почек в более высокую цель, связанную с вознесением тела. Существуют оверлеи, кодирующие человеческие тела для порабощения, установленные во время рождения, в записи последовательности трансдукции в теле проявления ядра или в Древе Жизни. Основной шаблон проявления сетки дерева имеет набор инструкций для контроля функций органов и желез на уровне каждого измерения, поскольку железы выделяют вещества и гормоны, которые позволяют человеческому сознанию двигаться быстрее между измерениями.
2. Ход работы.
В ходе работы вы освоите метод измерения основных характеристик источника тока, используя закон Ома для полной цепи, который связывает силу тока I в цепи, ЭДС источника тока , его внутреннее сопротивление r и сопротивление внешней цепи R соотношением:
В землях Соединенного Королевства ключи от пробуждения структур Альбиона скрыты, и они являются гигантскими спящими существами. Теги используются для руководства людьми на Земле для будущих временных линий для работы в рабских колониях или в различных галактических местах торговли людьми, которые контролируются этими внеземными коррумпированными конгломератами и группами драконов.
Группы Черного Солнца Ориона оставляли за собой право на некоторые человеческие тела, генетический материал и человеческое Древо Жизни, и именно поэтому они контролируют его. Благодаря этому им легче контролировать и контролировать информацию, связанную со структурой души и многомерной анатомией. Это драконовцы, которые воруют из духовных частей тела, а также из органов и желез.
. (9)
1 способ.
Схема экспериментальной установки показана на рисунке 1.
Внимательно изучите её. При разомкнутом ключе В источник замкнут на вольтметр, сопротивление которого много больше внутреннего сопротивления источника (r R ). В этом случае ток в цепи настолько мал, что можно пренебречь значением падения напряжения на внутреннем сопротивлении источника
, и ЭДС источника с пренебрежимо малой погрешностью равна напряжения на его зажимах , которое измеряется вольтметром, т. е.
. (10)
Таким образом, ЭДС источника определяется по показаниям вольтметра при разомкнутом ключе В.
Если ключ В замкнуть, вольтметр покажет падение напряжения на резисторе R :
. (11)
Тогда на основании равенств (9), (10) и (11) можно утверждать, что
(12)
Из формулы (12) видно, что для определения внутреннего сопротивления источника тока необходимо, кроме его ЭДС, знать силу тока в цепи и напряжение на резисторе R при замкнутом ключе.
Силу тока в цепи можно измерить при помощи амперметра. Проволочный резистор изготовлен из нихромовой проволоки и имеет сопротивление 5 Ом.
Соберите цепь по схеме, показанной на рисунке 3.
После того, как цепь будет собрана, необходимо поднять руку, позвать учителя, чтобы он проверил правильность сборки электрической цепи. И если цепь собрана правильно, то приступайте к выполнению работы.
При разомкнутом ключе В снимите показания вольтметра и занесите значение напряжения в таблицу 1. Затем замкните ключ В и опять снимите показания вольтметра, но уже и показания амперметра. Занесите значение напряжения и силы тока в таблицу 1.
Сформулируйте закон Ома для полной цепи.
Если бы мы не знали значения сопротивлений проволочных резисторов, то можно ли было бы использовать второй способ и что для этого надо сделать (может нужно, например, включить в цепь какой-нибудь прибор)?
Уметь собирать электрические цепи, используемые в работе.
Литература
Кабардин О. Ф.. Справ. Материалы: Учеб. Пособие для учащихся.-3-е изд.-М.:Просвещение,1991.-с.:150-151.
Справочник школьника. Физика/ Сост. Т. Фещенко, В. Вожегова.–М.: Филологическое об-щество «СЛОВО», ООО «Фирма» «Издательство АСТ», Центр гуманитарных наук при ф-те журна-листики МГУ им. М. В. Ломоносова, 1998. — с.: 124,500-501.
Самойленко П. И.. Физика (для нетехнических специальностей): Учебн. для общеобразоват. учреждений сред. Проф. Образования/ П. И.Самойленко, А. В. Сергеев.-2-е изд., стер.-М.: Издательский центр «Академия», 2003-с.: 181-182.
Цель работы: Научиться экспериментальным путем определять ЭДС, и внутреннее сопротивление источника тока.
Приборы и оборудование: Источники электрической энергии, амперметр (до 2А с делением до 0,1А), вольтметр (постоянного до 3А с делением до 0,3В), магазин (сопротивления до 10 Ом) ключ, соединительные провода.
ТЕОРИЯ:
Для поддержания тока в проводнике необходимо, чтобы разность потенциалов (напряжение) на его концах была неизменной. Для этого используется источник тока. Разность потенциалов на его полюсах образуется вследствие разделения зарядов на положительные и отрицательные. Работу по разделению зарядов выполняют сторонние силы (не электрического происхождения).
Величина, измеряемая работой, совершенной сторонними силами при перемещении единичного положительного электрического заряда внутри источника тока, называется электродвижущей силой источника тока (ЭДС) и выражается в вольтах.
Когда цепь замыкается, разделенные в источнике тока заряды образуют электрическое поле, которое перемещает заряды по внешней цепи; внутри же источника тока заряды движутся навстречу полю под действием сторонних сил. Таким образом, энергия, запасенная в источнике тока, расходуется на работу по перемещению заряда в цепи с внешним R и внутренним r сопротивлениями.
ХОД РАБОТЫ
1. Собрать электрическую цепь как показано на схеме.
2. Измерить ЭДС источника электрической энергии замкнув его на вольтметр (схема).
3. Измерить силу тока и падение напряжения на заданном сопротивлении.
№ | Е | U | I | R | r | rcр |
1. | ||||||
2. | ||||||
3. |
4. Вычислить внутреннее сопротивление по закону Ома для всей цепи.
5. Произвести опыты с другими сопротивлениями и вычислить внутреннее сопротивление элемента.
6. Вычислить среднее значение внутреннего сопротивления элемента.
7. Результаты всех измерений и вычислений записать в таблицу.
8. Найти абсолютную и относительную погрешность.
9. Сделать вывод.
КОНТРОЛЬНЫЕ ВОПРОСЫ
1. Укажите условия существования электрического тока в проводнике.
2. Какова роль источника электрической энергии в электрической цепи?
3. От чего зависит напряжение на зажимах источника электрической энергии?
ЛАБОРАТОРНАЯ РАБОТА № 7
ОПРЕДЕЛЕНИЕ ЭЛЕКТРОХИМИЧЕСКОГО ЭКВИВАЛЕНТА МЕДИ.
Цель работы : научиться на практике рассчитывать электрохимический эквивалент меди.
Оборудование: Весы с разновесом, амперметр, часы., источник электрической энергии, реостат, ключ, медные пластины (электроды), соединительные провода, электролитическая ванна с раствором медного купороса.
Теория
Процесс, при котором молекулы солей, кислот и щелочей при растворении в воде или других растворителях распадаются на заряженные частицы (ионы), называется электролитической диссоциацией, получившийся при этом раствор с положительными и отрицательными ионами называется электролитом.
Если в сосуд с электролитом поместить пластины (электроды), соединенные с зажимами источника тока (создать в электролите электрическое поле), то положительные ионы будут двигаться к катоду, а отрицательные — к аноду. Следовательно, в растворах кислот, солей и щелочей электрический заряд будет перемещаться вместе с частицами вещества. У электродов при этом происходит окислительно-восстановительные реакции, при которых на них выделяется вещество. Процесс прохождения электрического тока через электролит, сопровождающийся химическими реакциями называется электролизом.
Для электролиза справедлив закон Фарадея: масса выделившегося вещества на электроде прямо пропорциональна заряду, прошедшему через электролит:
где k-электрохимический эквивалент-количествовещества, выделенное при прохождении через электролит 1 Кл электричества. Измерив силу тока в цепи, время его прохождения и массу выделившегося на катоде вещества можно определить электрохимический эквивалент (1с выражается в кг/Кл).
где m-масса меди, выделившейся на катоде; I-сила тока в цепи; t- время пропускания тока в цепи.
Соберите электрическую цепь по схеме.
1. Одну из пластин, которая будет катодом, (если пластина мокрая, ее надо подсушить) тщательно взвесить с точностью до 10мг и записать результат в таблицу.
2. Вставить электрод в электролитическую ванну и составить электрическую цепь согласно схеме.
3. Отрегулировать реостатом ток, чтобы величина его не превышала 1А на 50см 2 погруженной части катодной пластины.
4. Замкнуть цепь на 15-20 минут.
5. Разомкнуть цепь, вынуть катодную пластинку, смыть с нее остатка раствора и высушить под рукосушителем.
6. Взвесить высушенную пластину с точностью до 10мг.
7. Значение тока, время опыта, увеличение в массе катодной пластину записать в таблицу и определить электрохимический эквивалент.
Оценка погрешностей.
.
Относительная погрешность:
.
, следовательно .
После этого дается результат в виде: .
Сравните полученный результат с табличным.
Контрольные вопросы.
1. Что такое электролитическая диссоциация, электролиз?
2. До каких пор будет происходить электролиз медного купороса, если оба электрода медные? Оба электрода угольные?
3. Быстрее или медленнее пойдет электролиз, если один из медных электродов заменить цинковым?
На концах проводника, а значит, и тока необходимо наличие сторонних сил неэлектрической природы, с помощью которых происходит разделение электрических зарядов .
Сторонними силами называются любые силы, действующие на электрически заряженные частицы в цепи, за исключением электростатических (т. е. кулоновских).
Сторонние силы приводят в движение заряженные частицы внут-ри всех источников тока: в генераторах, на электростанциях, в гальванических элементах, аккумуляторах и т. д.
При замыкании цепи создается электрическое поле во всех про-водниках цепи. Внутри источника тока заряды движутся под действием сторонних сил против кулоновских сил (электроны движут-ся от положительно заряженного электрода к отрицательному), а во всей остальной цепи их приводит а движение электрическое поле (см. рис. выше).
В источниках тока в процессе работы по разделению заряженных частиц происходит превращение разных видов энергии в электричес-кую. По типу преобразованной энергии различают следующие виды электродвижущей силы:
— электростатическая — в электрофорной машине, в которой происходит превращение механической энергии при трении в электрическую;
— термоэлектрическая — в термоэлементе — внутренняя энергия нагретого спая двух проволок, изготовленных из разных металлов, превращается в электрическую;
— фотоэлектрическая — в фотоэлементе. Здесь происходит превращение энергии света в элек-трическую: при освещении некоторых веществ, например, селена, оксида меди (I) , кремния наблюдается потеря отрицательного электрического заряда;
— химическая — в гальванических элементах, аккумуляторах и др. источниках, в которых происходит превращение химической энергии в электрическую.
Электродвижущая сила (ЭДС) — характеристика источников тока. Понятие ЭДС было введено Г. Омом в 1827 г. для цепей постоянного тока. В 1857 г. Кирхгофф определил ЭДС как работу сторонних сил при переносе единичного электрического заряда вдоль замкнутого контура:
ɛ = A ст /q ,
где ɛ — ЭДС источника тока, А ст — работа сторонних сил , q — количество перемещенного заряда.
Электродвижущую силу выражают в вольтах.
Можно говорить об электродвижущей силе на любом участке цепи. Это удельная работа сторонних сил (работа по перемещению единичного заряда) не во всем контуре, а только на данном участке.
Внутреннее сопротивление источника тока.
Пусть имеется простая замкнутая цепь, состоящая из источника тока (например, гальванического элемента, аккумулятора или генератора) и резистора с сопротивлением R . Ток в замкну-той цепи не прерывается нигде, следовательно, oн существует и внутри источника тока. Любой источник представляет собой некоторое сопротивление дли тока. Оно называется внутренним сопротивлением источника тока и обозначается буквой r .
В генераторе r — это сопротивление обмотки, в гальваническом элементе — сопротивление раствора электролита и электродов.
Таким образом, источник тока характеризуется величинами ЭДС и внутреннего сопротивлении, которые определяют его качество. Например, электростатические машины имеют очень большую ЭДС (до десятков тысяч вольт), но при этом их внутреннее сопротивление огромно (до со-тни Мом). Поэтому они непригодны для получения сильных токов. У гальванических элементов ЭДС всего лишь приблизительно 1 В, но зато и внутреннее сопротивление мало (приблизительно 1 Ом и меньше). Это позволяет с их помощью получать токи, измеряемые амперами.
Заглавная страница
КАТЕГОРИИ: Археология ТОП 10 на сайте Приготовление дезинфицирующих растворов различной концентрации Техника нижней прямой подачи мяча. Франко-прусская война (причины и последствия) Организация работы процедурного кабинета Смысловое и механическое запоминание, их место и роль в усвоении знаний Коммуникативные барьеры и пути их преодоления Обработка изделий медицинского назначения многократного применения Образцы текста публицистического стиля Четыре типа изменения баланса Задачи с ответами для Всероссийской олимпиады по праву Мы поможем в написании ваших работ! ЗНАЕТЕ ЛИ ВЫ? Влияние общества на человека Приготовление дезинфицирующих растворов различной концентрации Практические работы по географии для 6 класса Организация работы процедурного кабинета Изменения в неживой природе осенью Уборка процедурного кабинета Сольфеджио. Все правила по сольфеджио Балочные системы. Определение реакций опор и моментов защемления |
⇐ ПредыдущаяСтр 4 из 10Следующая ⇒ Как правило, заводские источники тока имеют очень маленькое внутреннее сопротивление, поэтому для получения зависимостей, отражающих рассмотренные выше энергетические соотношения, нужно последовательно с источником тока подключить сопротивление, имитирующее внутреннее сопротивление источника. Для этого можно использовать источник с e ~ 10 В, имитационное сопротивление ~ 100 ¸ 150 Ом. На рисунке клеммы «a» и «b» – выходные клеммы созданного «источника» с электродвижущей силой e и внутренним сопротивлением r (Рис.2). Для регистрации тока, протекающего по замкнутой цепи, и падения напряжения на нагрузке необходимо выбрать амперметр и вольтметр, исходя из e и максимального тока при нулевом сопротивлении нагрузки (тока короткого замыкания) . Собрать схему, выставить на источнике ЭДС и, изменяя внешнее сопротивление R (5 – 6 точек в диапазоне от 50 до 150 Ом), снять соответствующие показания амперметра и вольтметра. Построить график связи тока и напряжения. Экстраполировать полученную прямую до пересечения с осями напряжения и тока. Точка пересечения с осью напряжений даст значение ЭДС источника e, а точка пересечения с осью тока – значение тока короткого замыкания Iкз. Проверка энергетических соотношений в замкнутых цепях постоянного тока. По определенным в первом задании значениям e и Iкз оценить максимальное значение полной мощности Pmax, максимальное значение полезной мощности PR max, полную мощность при согласованной нагрузке. Снять зависимости тока и напряжения от сопротивления нагрузки, причем необходимо включить в это сопротивление и значения сопротивлений электроизмерительных приборов rA и rV: . Для получения необходимой точности при построении зависимости, нужно задавать такие значения внешнего сопротивления, чтобы примерно десять точек соответствовали условию , и около двадцати точек – условию . По полученным данным рассчитать и построить графики зависимостей полной мощности, полезной мощности и КПД от сопротивления нагрузки и от тока в цепи. Проанализировать полученные зависимости P(R), PR(R), h(R) и P(I), PR(I), h(I), обосновать ход зависимостей от тока и сделать вывод о выполнимости выведенных выше энергетических соотношений в замкнутых цепях постоянного тока. Построить зависимость потерь мощности от сопротивления внешней нагрузки DP(R) и тока в цепи DP(I). Вопросы к зачету по работе. – Почему на нагрузке выделяется энергия при протекании тока, сколько ее выделяется и на что она тратится? – Почем не вся мощность, вырабатываемая источником тока, может быть использована в качестве полезной мощности, и как уменьшить потери мощности? – Что означает «согласованная нагрузка» и чему равен КПД источника при согласовании нагрузки? – Почему согласованный режим работы является оптимальным? Лабораторная работа № 5 Изучение компенсационного метода измерения Цель работы. Изучить компенсационный метод измерения напряжений, токов и сопротивлений. Знания, необходимые для допуска к работе. – Закон Ома для цепи постоянного тока; – Правила Кирхгофа. Краткие сведения из теории. ЭДС источника тока нельзя точно измерить вольтметром (только если вольтметр не электростатической системы), потому что вольтметр имеет конечное сопротивление, а значит, при подключении его к источнику в замкнутом контуре потечет ток и часть напряжение будет падать на внутреннем сопротивлении источника. Вольтметр же измерит только внешнее падение напряжения, причем оно будет тем меньше, чем меньше сопротивление вольтметра по сравнению с сопротивлением источника. Компенсационный метод измерения позволяет измерять электрические параметры цепей с высокой точностью, как правило, не достижимой при использовании амперметров, вольтметров, гальванометров и т.д. Идея метода заключается в следующем. Падение напряжения на участке сопротивления R1, возникающее при протекании тока, включается навстречу подключенному параллельно неизвестному источнику ЭДС. При этом, если выполнится условие , гальванометр не покажет наличия тока в цепи неизвестного источника, и значит, не произойдет потерь напряжения на его внутреннем сопротивлении. А так как , то окончательная формула для определения неизвестного ЭДС выглядит так: . Добиться компенсации можно, меняя напряжение опорного источника U0 при неизменном положении движка реостата R, или при фиксированном напряжении U0 перемещая движок до наступления компенсации. Часто невозможно достичь желаемой точности в определении напряжения опорного источника U0 и параметров компенсирующей цепи R и R1. Тогда можно воспользоваться относительным методом, взяв в качестве эталонного ЭДС нормальный элемент. Его достоинством является высокая точность и стабильность выходной ЭДС. При комнатной температуре (~ 20°С) . Ясно, что данный метод компенсации позволяет измерять неизвестное ЭДС только при условии . В компенсационных схемах повышенной точности вместо измерительного сопротивления R используется набор декад образцовых сопротивлений, причем подключение их выполнено таким образом, чтобы сила тока в цепи опорного источника I0 оставалась неизменной. Тогда эти декады сопротивлений можно проградуировать по эталонному источнику в единицах напряжения и снимать показания сразу в этих единицах. Компенсационный метод позволяет измерять, правда косвенно, и ток, и сопротивление. При измерении сопротивления его и эталонное сопротивление включают последовательно. При протекании по ним тока I падение напряжения пропорционально величинам их сопротивлений , а почленно поделив эти выражения друг на друга, получаем формулу для определения неизвестного сопротивления: . По падению напряжения на эталонном сопротивлении можно точно определить силу тока Ix, протекающего в цепи, . Компенсационные методы могут быть использованы и для калибровки и градуировки электроизмерительных приборов. Практические задания ⇐ Предыдущая12345678910Следующая ⇒ Читайте также: Психологические особенности спортивного соревнования Приготовление дезинфицирующих растворов различной концентрации Занятость населения и рынок труда Социальный статус семьи и её типология |
Последнее изменение этой страницы: 2016-09-20; просмотров: 685; Нарушение авторского права страницы; Мы поможем в написании вашей работы! infopedia. su Все материалы представленные на сайте исключительно с целью ознакомления читателями и не преследуют коммерческих целей или нарушение авторских прав. Обратная связь — 176.9.44.166 (0.009 с.) |
Помощь студентам в учёбе от Людмилы Фирмаль
Здравствуйте!
Я, Людмила Анатольевна Фирмаль, бывший преподаватель математического факультета Дальневосточного государственного физико-технического института со стажем работы более 17 лет. На данный момент занимаюсь онлайн обучением и помощью по любыми предметам. У меня своя команда грамотных, сильных бывших преподавателей ВУЗов. Мы справимся с любой поставленной перед нами работой технического и гуманитарного плана. И не важно: она по объёму на две формулы или огромная сложно структурированная на 125 страниц! Нам по силам всё, поэтому не стесняйтесь, присылайте.
Срок выполнения разный: возможно онлайн (сразу пишите и сразу помогаю), а если у Вас что-то сложное – то от двух до пяти дней.
Для качественного оформления работы обязательно нужны методические указания и, желательно, лекции. Также я провожу онлайн-занятия и занятия в аудитории для студентов, чтобы дать им более качественные знания.
Моё видео:
Вам нужно написать сообщение в Telegram . После этого я оценю Ваш заказ и укажу срок выполнения. Если условия Вас устроят, Вы оплатите, и преподаватель, который ответственен за заказ, начнёт выполнение и в согласованный срок или, возможно, раньше срока Вы получите файл заказа в личные сообщения.
Сколько может стоить заказ?Стоимость заказа зависит от задания и требований Вашего учебного заведения. На цену влияют: сложность, количество заданий и срок выполнения. Поэтому для оценки стоимости заказа максимально качественно сфотографируйте или пришлите файл задания, при необходимости загружайте поясняющие фотографии лекций, файлы методичек, указывайте свой вариант.
Какой срок выполнения заказа?Минимальный срок выполнения заказа составляет 2-4 дня, но помните, срочные задания оцениваются дороже.
Как оплатить заказ?Сначала пришлите задание, я оценю, после вышлю Вам форму оплаты, в которой можно оплатить с баланса мобильного телефона, картой Visa и MasterCard, apple pay, google pay.
Какие гарантии и вы исправляете ошибки?В течение 1 года с момента получения Вами заказа действует гарантия. В течении 1 года я и моя команда исправим любые ошибки в заказе.
Качественно сфотографируйте задание, или если у вас файлы, то прикрепите методички, лекции, примеры решения, и в сообщении напишите дополнительные пояснения, для того, чтобы я сразу поняла, что требуется и не уточняла у вас. Присланное качественное задание моментально изучается и оценивается.
Теперь напишите мне в Telegram или почту и прикрепите задания, методички и лекции с примерами решения, и укажите сроки выполнения. Я и моя команда изучим внимательно задание и сообщим цену.
Если цена Вас устроит, то я вышлю Вам форму оплаты, в которой можно оплатить с баланса мобильного телефона, картой Visa и MasterCard, apple pay, google pay.
Мы приступим к выполнению, соблюдая указанные сроки и требования. 80% заказов сдаются раньше срока.
После выполнения отправлю Вам заказ в чат, если у Вас будут вопросы по заказу – подробно объясню. Гарантия 1 год. В течении 1 года я и моя команда исправим любые ошибки в заказе.
youtube.com/embed/FTtp-vAv3lI» frameborder=»0″ allow=»accelerometer; autoplay; encrypted-media; gyroscope; picture-in-picture» allowfullscreen=»»/>
youtube.com/embed/7LD4euDcivY» frameborder=»0″ allow=»accelerometer; autoplay; encrypted-media; gyroscope; picture-in-picture» allowfullscreen=»»/>
youtube.com/embed/ZfNUaigadEY» frameborder=»0″ allow=»accelerometer; autoplay; encrypted-media; gyroscope; picture-in-picture» allowfullscreen=»»/>
Можете смело обращаться к нам, мы вас не подведем. Ошибки бывают у всех, мы готовы дорабатывать бесплатно и в сжатые сроки, а если у вас появятся вопросы, готовы на них ответить.
В заключение хочу сказать: если Вы выберете меня для помощи на учебно-образовательном пути, у вас останутся только приятные впечатления от работы и от полученного результата!
Жду ваших заказов!
С уважением
Пользовательское соглашение
Политика конфиденциальности
Урок 31. Лабораторная работа № 08. Измерение ЭДС и внутреннего сопротивления источника тока.
Лабораторная работа № 8
Тема: «Определение электродвижущей силы и внутреннего сопротивления источника тока».
Цель: научиться определять электродвижущую силу и внутреннее сопротивление источника электрической энергии.
Оборудование: 1. Амперметр лабораторный;
2. Источник электрической энергии;
3. Соединительные провода,
4. Набор сопротивлений 2 Ом и 4 Ом;
5. Переключатель однополюсный; ключ.
Теория.
Возникновение разности потенциалов на полюсах любого источника является результатом разделения в нем положительных и отрицательных зарядов. Это разделение происходит благодаря работе, совершаемой сторонними силами.
Силы неэлектрического происхождения, действующие на свободные носители заряда со стороны источников тока, называются сторонними силами.
При перемещении электрических зарядов по цепи постоянного тока сторонние силы, действующие внутри источников тока, совершают работу.
Физическая величина, равная отношению работы Aст сторонних сил при перемещении заряда q внутри источника тока к величине этого заряда, называется электродвижущей силой источника (ЭДС):
ЭДС определяется работой, совершаемой сторонними силами при перемещении единичного положительного заряда.
Электродвижущая сила, как и разность потенциалов, измеряется в вольтах [В].
Чтобы измерить ЭДС источника, надо присоединить к нему вольтметр при разомкнутой цепи.
Источник тока является проводником и всегда имеет некоторое сопротивление, поэтому ток выделяет в нем тепло. Это сопротивление называют внутренним сопротивлением источника и обозначают r.
Если цепь разомкнута, то работа сторонних сил превращается в потенциальную энергию источника тока. При замкнутой цепи эта потенциальная энергия расходуется на работу по перемещению зарядов во внешней цепи с сопротивлением R и во внутренней части цепи с сопротивлением r , т.е. ε = IR + Ir.
Если цепь состоит из внешней части сопротивлением R и внутренней сопротивлением r, то, согласно закону сохранения энергии, ЭДС источника будет равна сумме напряжений на внешнем и внутреннем участках цепи, т.к. при перемещении по замкнутой цепи заряд возвращается в исходное положение , где IR – напряжение на внешнем участке цепи, а Ir — напряжение на внутреннем участке цепи.
Таким образом, для участка цепи, содержащего ЭДС:
Эта формула выражает закон Ома для полной цепи: сила тока в полной цепи прямо пропорциональна электродвижущей силе источника и обратно пропорциональна сумме сопротивлений внешнего и внутреннего участков цепи.
ε и r можно определить опытным путем.
Часто источники электрической энергии соединяют между собой для питания цепи. Соединение источников в батарею может быть последовательным и параллельным.
При последовательном соединении два соседних источника соединяются разноименными полюсами.
Т.е., для последовательного соединения аккумуляторов, к ″плюсу″ электрической схемы подключают положительную клемму первого аккумулятора. К его отрицательной клемме подключают положительную клемму второго аккумулятора и т.д. Отрицательную клемму последнего аккумулятора подключают к ″минусу″ электрической схемы.
Получившаяся при последовательном соединении аккумуляторная батарея имеет ту же емкость, что и у одиночного аккумулятора, а напряжение такой аккумуляторной батареи равно сумме напряжений входящих в нее аккумуляторов. Т.е. если аккумуляторы имеют одинаковые напряжения, то напряжение батареи равно напряжению одного аккумулятора, умноженному на количество аккумуляторов в аккумуляторной батарее.
1. ЭДС батареи равна сумме ЭДС отдельных источников ε= ε1 + ε2 + ε3
2. Общее сопротивление батареи источников равно сумме внутренних сопротивлений отдельных источников rбатареи= r1 + r2 + r3
Если в батарею соединены n одинаковых источников, то ЭДС батареи ε= nε1, а сопротивление rбатареи= nr1
3. Сила тока в такой цепи по закону Ома
При параллельном соединении соединяют между собой все положительные и все отрицательные полюсы двух или n источников.
Т.е., при параллельном соединении, аккумуляторы соединяют так, чтобы положительные клеммы всех аккумуляторов были подключены к одной точке электрической схемы (″плюсу″), а отрицательные клеммы всех аккумуляторов были подключены к другой точке схемы (″минусу″).
Параллельно соединяют только источники с одинаковой ЭДС. Получившаяся при параллельном соединении аккумуляторная батарея имеет то же напряжение, что и у одиночного аккумулятора, а емкость такой аккумуляторной батареи равна сумме емкостей входящих в нее аккумуляторов. Т.е. если аккумуляторы имеют одинаковые емкости, то емкость аккумуляторной батареи равна емкости одного аккумулятора, умноженной на количество аккумуляторов в батарее.
1. ЭДС батареи одинаковых источников равна ЭДС одного источника. ε= ε1= ε2 = ε3
2. Сопротивление батареи меньше, чем сопротивление одного источника rбатареи= r1/n
3. Сила тока в такой цепи по закону Ома
Электрическая энергия, накопленная в аккумуляторной батарее равна сумме энергий отдельных аккумуляторов (произведению энергий отдельных аккумуляторов, если аккумуляторы одинаковые), независимо от того, как соединены аккумуляторы — параллельно или последовательно.
Внутреннее сопротивление аккумуляторов, изготовленных по одной технологии, примерно обратно пропорционально емкости аккумулятора. Поэтому т.к.при параллельном соединении емкость аккумуляторной батареи равна сумме емкостей входящих в нее аккумуляторов, т.е увеличивается, то внутреннее сопротивление уменьшается.
Ход работы.
1. Начертите таблицу:
№ опыта |
Источник электрической энергии ВУП, В |
1-й отсчет |
2-й отсчет |
Э.Д.С. ε , В |
Внутреннее сопротивление, r , Ом |
||
R1, Ом |
Сила тока I1 , А |
R2, Ом |
Сила тока I2 , А |
||||
1
|
1 |
1 |
|
2 |
|
|
|
2. Рассмотрите шкалу амперметра и определите цену одного деления.
3. Составьте электрическую цепь по схеме, изображенной на рисунке 1. Переключатель поставить в среднее положение.
Рисунок 1.
4. Замкнуть цепь, введя меньшее сопротивление R1. Записать величину силы тока I1. Разомкнуть цепь.
5. Замкнуть цепь, введя большее сопротивление R2. Записать величину силы тока I2. Разомкнуть цепь.
6. Вычислить значение ЭДС и внутреннего сопротивления источника электрической энергии.
Закон Ома для полной цепи для каждого случая: и
Отсюда получим формулы для вычисления ε и r:
7. Результаты всех измерений и вычислений запишите в таблицу.
8. Сделайте вывод.
9. Ответьте на контрольные вопросы.
КОНТРОЛЬНЫЕ ВОПРОСЫ.
1. Раскройте физический смысл понятия «электродвижущая сила источника тока».
2. Определить сопротивление внешнего участка цепи, пользуясь результатами полученных измерений и законом Ома для полной цепи.
3. Объяснить, почему внутреннее сопротивление возрастает при последовательном соединении аккумуляторов и уменьшается при параллельном в сравнении с сопротивлением r0 одного аккумулятора.
4. В каком случае вольтметр, включенный на зажимы генератора, показывает ЭДС генератора и в каком случае напряжение на концах внешнего участка цепи? Можно ли это напряжение считать также и напряжением на концах внутреннего участка цепи?
Вариант выполнения измерений.
Опыт 1. Сопротивление R1=2 Ом, сила тока I1=1,3 А.
Сопротивление R2=4 Ом, сила тока I2=0,7 А.
ЛР №3
Лабораторная работа № 3
ЭДС и внутреннее сопротивление источников постоянного тока. Закон Ома для полной цепи.
Цель: определить внутреннее сопротивление источника тока и его ЭДС.
1. Краткое теоретическое описание
Электрический ток в проводниках вызывают так называемые источники постоянного тока. Силы, вызывающие перемещение электрических зарядов внутри источника постоянного тока против направления действия сил электростатического поля, называются сторонними силами. Отношение работы Астор., совершаемой сторонними силами по перемещению заряда Q вдоль цепи, к значению этого заряда называется электродвижущей силой источника (ЭДС):
(1)
Электродвижущая сила выражается в тех же единицах, что и напряжение или разность потенциалов, т.е. в Вольтах.
Работа – эта мера превращения энергии из одного вида в другой. Следовательно, в источнике сторонняя энергия преобразуется в энергию электрического поля
W = Q (2)
При движении заряда Q на внешнем участке цепи преобразуется энергия стационарного поля, созданного и поддерживаемого источником:
W1 = U Q , (3)
а на внутреннем участке:
W2 = Uвн. Q (4)
По закону сохранения энергии
W = W1 + W2 или Q =U Q + Uвн.Q (5)
Сократив на Q, получим:
= Uвн. + U (6)
т.е. электродвижущая сила источника равна сумме напряжений на внешнем и внутреннем участке цепи.
При разомкнутой цепи Uвн.= 0, то
= U (7)
Подставив в равенство (6) выражения для U и Uвн. по закону Ома для участка цепи
U = I R; Uвн. = I r,
получим:
= I R + I r = I (R + r) (8)
Отсюда
(9)
Таким образом, сила тока в цепи равна отношению электродвижущей силы источника к сумме сопротивлений внешнего и внутреннего участков цепи. Это закон Ома для полной цепи. В формулу (9) входит внутреннее сопротивление r.
Рис.1
Пусть известны значения сил токов I1 и I2 и падения напряжений на реостате U1 и U2 (см. рис.1.). Для ЭДС можно записать:
= I1(R1 + r) и = I2(R2 + r) (10)
Приравнивая правые части этих двух равенств, получим
I1(R1 + r) = I2(R2 + r)
или
I1R1 + I1r = I2 R2 + I2r
I1r – I2r = I2 R2 — I1R1
Т.к. I1 R1 = U1 и I2 R2 = U2, то можно последнее равенство записать так
r (I1 – I2) = U2 – U1 ,
откуда
(11)
2. Порядок выполнения работы
2.1. Соберите цепь по схеме, изображенной на рисунке 1. Установите сопротивление реостата 7 Ом, ЭДС батарейки 1,5 В, внутреннее сопротивление батарейки 3 Ом.
2.2. При помощи мультиметра определите напряжение на батарейке при разомкнутом ключе. Это и будет ЭДС батарейки в соответствии с формулой (7).
2.3. Замкните ключ и измерьте силу тока и напряжение на реостате. Запишите показания приборов.
2.4. Измените сопротивление реостата и запишите другие значения силы тока и напряжения.
2.5. Повторите измерения силы тока и напряжения для 6 различных положений ползунка реостата и запишите полученные значения в таблицу.
2.6. Рассчитайте внутреннее сопротивление по формуле (11).
2.7. Определите абсолютную и относительную погрешность измерения ЭДС и внутреннего сопротивления батарейки.
V = 1.49 B
V=0.950 B I=0.33 A
Вывод: Научился определять внутреннее сопротивление источника тока и его ЭДС
3. Контрольные вопросы
3.1. Сформулируйте закон Ома для полной цепи.
3.2. Чему равно ЭДС источника при разомкнутой цепи?
3.3. Чем обусловлено внутреннее сопротивление источника тока?
3.4. Чем определяется сила тока короткого замыкания батарейки?
Ответы:
3.1 Сила тока пропорциональна ЭДС и обратно пропорциональна сумме внутреннего и внешнего сопротивлений.
3.2 В таком случае ЭДС равна напряжению
3.3 Отношению изменения силы тока к изменению напряжения
3.4 Отношением ЭДС в внутреннему сопротивлению
ЭДС и внутреннее сопротивление: измерения и взаимосвязь между
иЭлектродвижущая сила , известная как ЭДС, представляет собой конечную разность потенциалов источника при отсутствии тока. Внутреннее сопротивление — это сопротивление протеканию тока внутри самого источника. Но, что важно, как мы вычисляем эти значения? Давай выясним.
Что такое ЭДС в электрических цепях?
Все источники напряжения создают разность потенциалов, обеспечивая ток при подключении к цепи с сопротивлением. Эта разность потенциалов создает электрическое поле, которое действует на заряды как сила, заставляя течь ток.
Несмотря на свое название, ЭДС не совсем сила. На самом деле это уникальный вид разности потенциалов и измеряется в вольтах (В).
ЭДС — это разность потенциалов источника, когда через него не протекает ток .
Мы также можем определить ЭДС как работу W, выполненную на единицу заряда Q, что дает нам следующее уравнение:
Все источники напряжения производят ЭДС, Unsplash
Представьте себе батарею.
- Если батарея подает ток, напряжение на клеммах батареи в раз меньше ЭДС . По мере разрядки батареи этот уровень напряжения начинает снижаться.
- Когда батарея полностью разряжена и, следовательно, не подает ток, напряжение на клеммах батареи будет равно ЭДС.
Как рассчитать ЭДС?
Мы также можем рассчитать ЭДС (ε) с помощью следующего уравнения:
E обозначает электрическую энергию в джоулях (Дж), а Q — заряд в кулонах (Кл).
В этом уравнении разность потенциалов называется конечной разностью потенциалов . Будет равна ЭДС, если нет внутреннего сопротивления. Однако это не относится к реальным источникам питания, поскольку всегда существует внутреннее сопротивление. Lost v olts относятся к энергии, затраченной на кулон при преодолении внутреннего сопротивления.
Мы знаем, что закон сохранения энергии проявляется в электрических цепях, а также в случаях, когда имеется внутреннее сопротивление.
Уравнение сохранения энергии с внутренним сопротивлением, Oğulcan Tezcan – StudySmarter Originals
Потерянные вольты – это название энергии, затраченной на кулон при преодолении внутреннего сопротивления. Кроме того, обязательно ознакомьтесь с нашим объяснением по энергосбережению.
Что такое внутреннее сопротивление в электрических цепях?
Мы уже знаем, что сопротивление нагрузки (также известное как внешнее сопротивление) представляет собой полное сопротивление компонентов внешней электрической цепи. С другой стороны, внутреннее сопротивление представляет собой сопротивление в источнике питания, которое сопротивляется протеканию тока. Обычно это приводит к тому, что источник питания выделяет тепло.
- Сопротивление нагрузки = общее сопротивление компонентов внешней электрической цепи.
- Внутреннее сопротивление = сопротивление внутри источника питания, которое сопротивляется протеканию тока.
Измерение внутреннего сопротивления
Закон о Ом
Из закона Ома мы знаем, что
, где В — это напряжение в Volts, I . внешнее сопротивление в омах.
Внутреннее сопротивление
Если мы включим внутреннее сопротивление, общее сопротивление будет R+r, где внутреннее сопротивление показано как r, а напряжение может быть выражено как ЭДС (ε).
Если раскрыть скобки, получится
, где I⋅R — разность потенциалов на клеммах в вольтах, а I⋅r — потерянные вольты (также измеряемые в вольтах).
Теперь мы можем преобразовать уравнение как
, где V R — разность потенциалов на клеммах, а V r — потерянные вольты.
Зависимость между разностью потенциалов на клеммах и потерями вольт
Здесь представлена зависимость между разностью потенциалов на клеммах и потерями вольт. Из уравнения видно, что если есть нет внутреннего сопротивления (поэтому нет потерянных вольт), оконечное сопротивление будет равно ЭДС .
Принципиальная схема, показывающая внутреннее сопротивление и сопротивление нагрузки, Oğulcan Tezcan – StudySmarter Originals
Внутреннее сопротивление (r) имеет сложные характеристики. Давайте снова посмотрим на наш пример с аккумулятором. По мере разрядки батареи ее внутреннее сопротивление возрастает. А что еще влияет на внутреннее сопротивление? Вот некоторые факторы:
- Размер источника напряжения.
- Сколько и как долго он использовался.
- Величина и направление тока через источник напряжения.
Приведите примеры расчета ЭДС и внутреннего сопротивления?
Расчет внутреннего сопротивления источника является важным фактором для достижения оптимальной эффективности и обеспечения максимальной мощности источника для электрической цепи. Вот несколько примеров расчета различных величин с внутренним сопротивлением.
Помните, что R означает сопротивление нагрузки, а r — внутреннее сопротивление.
Аккумулятор имеет ЭДС 0,28 В и внутреннее сопротивление 0,65 Ом. Рассчитайте разность потенциалов на клеммах, когда ток, протекающий через батарею, равен 7,8 мА.
Решение
В вопросе указаны ЭДС (ε), внутреннее сопротивление (r) и ток (I), протекающий через батарею. Давайте поместим их в уравнение конечной разности потенциалов (V R ).
Через ячейку протекает ток 0,45 А с внутренним сопротивлением 0,25 Ом. Найдите энергию, затрачиваемую в секунду на внутреннее сопротивление в джоулях.
Решение
Мы знаем, что
где P — мощность в ваттах, I — сила тока в амперах, а R — сопротивление в омах.
Поскольку в вопросе речь идет о потере энергии в секунду, мы используем уравнение мощности, потому что мощность — это энергия в секунду. Мы также можем указать внутреннее сопротивление r для сопротивления в уравнении.
Аккумулятор имеет ЭДС 0,35 В. Ток, протекающий через аккумулятор, равен 0,03 А, а сопротивление нагрузки равно 1,2 Ом. Найдите внутреннее сопротивление батареи.
Решение
Значение ЭДС (ε) батареи, ток (I), протекающий через батарею, и сопротивление нагрузки (R) даются в вопросе. Вот правильное уравнение для определения внутреннего сопротивления (r):
Подставим данные переменные в уравнение:
Если мы решим уравнение для r, мы получим
ЭДС и внутреннее сопротивление — основные выводы
- Электродвижущая сила — это не совсем сила: это уникальный вид разности потенциалов, измеряемый в вольтах. .
- При отсутствии тока напряжение на клеммах источника напряжения будет равно ЭДС.
- Потерянные вольты — это название энергии, затрачиваемой на кулон при преодолении внутреннего сопротивления.
- Внутреннее сопротивление — это сопротивление внутри источника питания, которое сопротивляется протеканию тока и обычно заставляет источник питания выделять тепло.
- Внутреннее сопротивление источника напряжения зависит от множества условий, в том числе от степени его использования, размера источника напряжения, величины и направления тока, протекающего через источник напряжения.
домашнее задание и упражнения — Определение тока с помощью ЭДС и внутреннего сопротивления
спросил
Изменено 6 месяцев назад
Просмотрено 35 тысяч раз
$\begingroup$
В чем именно разница между внутренним сопротивлением и сопротивлением?
Это всплыло в связи с домашним заданием, которое мне дали:
Схема, показанная на рисунке, содержит две батареи, каждая с ЭДС и внутреннее сопротивление, и два резистора.
Мне нужно найти величину тока в этой цепи.
Кажется, я должен использовать уравнение: $I = \frac E{(R + r)}$
, где $E = 24,0 В, R = 17 \Omega$.
Итак, как определить внутреннее сопротивление.
- домашние задания и упражнения
- электрические цепи
- напряжение
- электрическое сопротивление
- электрический ток
$\endgroup$
1
$\begingroup$
какая именно разница между внутренним сопротивлением и сопротивление?
Идеальный источник напряжения может подавать неограниченный ток во внешнюю цепь, чтобы поддерживать напряжение источника.
Но идеальных источников напряжения не бывает, т. е. все реальные источники напряжения имеют некоторый максимальный ток, подаваемый в короткое замыкание.
Это моделируется путем последовательного включения резистора с идеальным источником напряжения, и это сопротивление является «внутренним сопротивлением» или «сопротивлением источника».
Ясно, что максимальный ток от такого источника: источник, когда источник не обеспечивает тока.
Это связано с концепциями эквивалентных схем Тевенина и Нортона.
(Примечание: исходная ссылка не работает — теперь ссылка на самый последний архив)
$\endgroup$
2
$\begingroup$
Внутреннее сопротивление обычно относится к сопротивлению батарей. Таким образом, $1,6$ $\Omega$ и $1,4$ $\Omega$ относятся к батареям $16,0 V$ и $8,0 V$ соответственно.
Отличается от сопротивления тем, что сопротивление связано с резисторами в цепи.
$R = 5,0 + 9,0 = 14 \Омега$
$r = 1,6 + 1,4 = 3 \Omega$
Общее сопротивление = $R + r = 17 \Omega$.
Затем используйте $I = \mathcal E / (R+r)$
$I = (16,0 — 8,0)/(17) = 8/17 A$.
Направление тока от положительной полярности $16 В$ к отрицательной полярности.
Почему $\mathcal E = (16,0 — 8,0)$? Это следует из законов Кирхгофа.
На ваш вопрос «Как мне найти внутреннее сопротивление» в том виде, как был задан вопрос, внутреннее сопротивление $r$ вычисляется из желтой части рисунка. Эта часть содержит как символ батареи, так и символ сопротивления. Они относятся к аккумуляторам.
$\endgroup$
$\begingroup$
Эй, вы ошибаетесь, эквивалентная ЭДС (напряжение) системы будет 16–8 В, потому что обе противоположные полюса обращены друг к другу, поэтому их ток будет чистым (от -ve до +ve) в соответствии с ячейкой. большей ЭДС (ячейка 16 В). Тогда ваше общее сопротивление составляет $ 5 + 1,6 + 1,4 = 8 омега $ $ (все последовательно). $$I(ток) = E(эдс или напряжение)/R(сопротивление) = 8/8 = 1 А(ампер)$$
$\endgroup$
Твой ответ
Зарегистрируйтесь или войдите в систему
Зарегистрируйтесь с помощью Google
Зарегистрироваться через Facebook
Зарегистрируйтесь, используя электронную почту и пароль
Опубликовать как гость
Электронная почта
Требуется, но не отображается
Опубликовать как гость
Электронная почта
Требуется, но не отображается
Нажимая «Опубликовать свой ответ», вы соглашаетесь с нашими условиями обслуживания, политикой конфиденциальности и политикой использования файлов cookie
.Изменения ЭДС и внутреннего сопротивления источника тока. Электродвижущая сила
Сделали вывод, что для поддержания постоянного тока в замкнутой цепи необходимо включить в нее источник тока. Подчеркнем, что задача источника состоит не в том, чтобы подавать заряды в электрическую цепь (этих зарядов достаточно в проводниках), а в том, чтобы заставить их двигаться, совершать работу по перемещению зарядов против сил электрического поля. Основная характеристика источника электродвижущая сила 1 (ЭДС) – это работа, совершаемая внешними силами для перемещения единичного положительного заряда
Единицей измерения ЭДС в системе единиц СИ является вольт. ЭДС источника равна 1 вольту, если он совершает работу в 1 Дж при перемещении заряда в 1 кулон
Для обозначения источников тока на электрических цепях применяют специальное обозначение (рис. 397).
рис. 397
Электростатическое поле совершает положительную работу по перемещению положительного заряда в направлении уменьшения потенциала поля. Источник тока проводит разделение электрических зарядов — на одном полюсе накапливаются положительные заряды, на другом — отрицательные. Напряжённость электрического поля в источнике направлена от положительного полюса к отрицательному, поэтому работа электрического поля по перемещению положительного заряда будет положительной при его движении от «плюса» к «минусу». Работа внешних сил, наоборот, положительна, если положительные заряды движутся от отрицательного полюса к положительному, то есть от «минуса» к «плюсу».
В этом принципиальное различие понятий разности потенциалов и ЭДС, о котором нужно всегда помнить.
Таким образом, электродвижущую силу источника можно считать алгебраической величиной, знак которой («плюс» или «минус») зависит от направления тока. В схеме, показанной на рис. 398,
рис. 398
вне источника (во внешней цепи) ток течет 2 от «плюса» источника к «минусу», внутри источника от «минуса» к «плюсу». В этом случае как силы внешнего источника, так и электростатические силы во внешней цепи совершают положительную работу.
Если на каком-то участке электрической цепи помимо электростатических сил действуют еще и сторонние силы, то на движение зарядов «работают» как электростатические, так и сторонние силы. Суммарная работа электростатических и внешних сил по перемещению одиночного положительного заряда называется электрическим напряжением на участке цепи
В случае отсутствия внешних сил электрическое напряжение совпадает с разностью потенциалов электрического поля.
Поясним определение напряжения и знака ЭДС на простом примере. Пусть на участке цепи, по которому протекает электрический ток, имеется источник внешних сил и резистор (рис. 39).9).
рис. 399
Для определенности будем считать, что φ o > φ 1 , то есть электрический ток направлен из точки 0 до точки 1 . При подключении источника, как показано на рис. 399 а, внешние силы источника совершают положительную работу, поэтому соотношение (2) в этом случае можно записать как
При повторном включении источника (рис. 399 б), заряды внутри него движутся против внешних сил, поэтому работа последних отрицательна. Фактически силы внешнего электрического поля преодолевают внешние силы. Поэтому в этом случае рассматриваемое соотношение (2) имеет вид
При утечке электрического тока на участке цепи, имеющем электрическое сопротивление, необходимо совершить работу по преодолению сил сопротивления. Для единичного положительного заряда эта работа по закону Ома равна произведению IR = U что, конечно же, совпадает с напряжением на этом участке.
Заряженные частицы (как электроны, так и ионы) внутри источника движутся в некоторой среде, поэтому со стороны среды на них также действуют тормозящие силы, которые также необходимо преодолевать. Заряженные частицы преодолевают силы сопротивления за счет действия внешних сил (если ток в источнике направлен от «плюса» к «минусу») или за счет электростатических сил (если ток направлен от «минуса» к «плюсу») . Очевидно, что работа по преодолению этих сил не зависит от направления движения, так как силы сопротивления всегда направлены в сторону, противоположную скорости движения частиц. Так как силы сопротивления пропорциональны средней скорости движения частиц, то и работа по их преодолению пропорциональна скорости движения, следовательно, и силе тока. Таким образом, мы можем ввести еще одну характеристику источника — его внутреннее сопротивление r , аналогично обычному электрическому сопротивлению. Работа по преодолению сил сопротивления при перемещении единичного положительного заряда между полюсами источника равна A/q = Ir . Подчеркнем еще раз, что эта работа не зависит от направления тока в источнике.
1 Название этой физической величины неудачное — значит, электродвижущая сила есть работа, а не сила в обычном механическом понимании. Но этот термин так устоялся, что «не в нашей власти» его изменить. Кстати, сила тока не равна механической силе! Не говоря уже о таких понятиях, как «стойкость», «сила воли», «божественная сила» и т. д.
2 Напомним, что направление движения положительных зарядов принимается за направление движения электрического тока.
На концах проводника, а значит и тока, необходимо наличие внешних сил неэлектрического характера, с помощью которых происходит разделение электрических зарядов.
Сторонние силы называются любые силы, действующие на электрически заряженные частицы в цепи, за исключением электростатических (т. е. кулоновских).
Сторонние силы приводят в движение заряженные частицы внутри всех источников тока: в генераторах, на электростанциях, в гальванических элементах, батареях и т. д.
При замыкании цепи во всех проводниках цепи создается электрическое поле. Внутри источника тока заряды движутся под действием внешних сил против кулоновских сил (электроны движутся от положительно заряженного электрода к отрицательному), а в остальной части цепи их приводит в движение электрическое поле (см. рис. выше ).
В источниках тока в процессе работы по разделению заряженных частиц происходит преобразование различных видов энергии в электричество. По виду преобразуемой энергии различают следующие виды электродвижущей силы:
— электростатическая — в электрофорной машине, в которой происходит преобразование механической энергии при трении в электрическую;
— термоэлектрический — в термоэлементе — внутренняя энергия нагретого соединения двух проводов из разных металлов превращается в электрическое;
— фотоэлектрический — в фотоэлементе. Здесь энергия света превращается в электрическую: при освещении некоторых веществ, например селена, оксида меди (I), кремния, происходит потеря отрицательного электрического заряда;
— химические — в гальванических элементах, батареях и других источниках, в которых химическая энергия преобразуется в электрическую.
Электродвижущая сила (ЭДС) — характеристика источников тока. Понятие ЭДС было введено Г. Омом в 1827 г. для цепей постоянного тока. В 1857 г. Кирхгоф определил ЭДС как работу внешних сил при переносе единицы электрического заряда по замкнутой цепи:
ɛ = A st/q ,
где ɛ — ЭДС источника тока, А ст — работа внешних сил, q — количество переданного заряда.
Электродвижущая сила выражается в вольтах.
Мы можем говорить об электродвижущей силе в любой части цепи. Это удельная работа внешних сил (работа перемещения единичного заряда) не во всей цепи, а только на этом участке.
Внутреннее сопротивление источника тока.
Пусть имеется простая замкнутая цепь, состоящая из источника тока (например, гальванического элемента, батареи или генератора) и резистора с сопротивлением R . Ток в замкнутой цепи нигде не прерывается, следовательно, он существует и внутри источника тока. Любой источник представляет некоторое сопротивление току. Он называется , внутреннее сопротивление источника тока и маркируется буквой r .
В генераторе р — это сопротивление обмотки, в гальваническом элементе — сопротивление раствора электролита и электродов.
Таким образом, источник тока характеризуется значениями ЭДС и внутреннего сопротивления, которые определяют его качество. Например, электростатические машины имеют очень большую ЭДС (до десятков тысяч вольт), но в то же время их внутреннее сопротивление огромно (до сотен МОм). Поэтому они непригодны для приема больших токов. У гальванических элементов ЭДС всего около 1 В, но внутреннее сопротивление тоже невелико (около 1 Ом и меньше). Это позволяет им получать токи, измеряемые в амперах.
Цель: № для изучения методики измерения ЭДС и внутреннего сопротивления источника тока с помощью амперметра и вольтметра.
Оборудование: металлический планшет, источник тока, амперметр, вольтметр, резистор, ключ, зажимы, соединительные провода.
Для измерения ЭДС и внутреннего сопротивления источников тока соберите электрическую цепь, схема которой представлена на рисунке 1.
К источнику тока подключают последовательно соединенные амперметр, сопротивление и ключ. Кроме того, вольтметр также подключается непосредственно к выходным гнездам источника.
ЭДС измеряется по показаниям вольтметра при открытом ключе. Эта методика определения ЭДС основана на следствии закона Ома для полной цепи, согласно которому при бесконечно большом сопротивлении внешней цепи напряжение на зажимах источника равно его ЭДС. (См. параграф «Закон Ома для полной цепи» учебника «Физика 10»).
Для определения внутреннего сопротивления источника ключ К замыкается. При этом в схеме можно условно выделить два участка: внешний (тот, который подключен к источнику) и внутренний (тот, что находится внутри источника тока). Так как ЭДС источника равна сумме падений напряжения на внутреннем и внешнем участках цепи:
ε = У р +У Р , затем У р = ε -У Р (1)
По закону Ома для участка цепи U r = I · р(2). Подставляя равенство (2) в (1), получаем:
I · р = ε — У Р , откуда r = ( ε — У Р )/ Дж
Следовательно, чтобы узнать внутреннее сопротивление источника тока, необходимо сначала определить его ЭДС, затем замкнуть ключ и измерить падение напряжения на внешнем сопротивлении, а также силу тока в этом.
Рабочий процесс
1. Подготовить таблицу для записи результатов измерений и расчетов:
ε ,в | У р , Б | и,а | р , Ом |
Нарисуйте в тетради схему измерения ЭДС и внутреннего сопротивления источника.
После проверки цепи соберите электрическую цепь. Откройте ключ.
Измерение источника значения ЭДС.
Замкните выключатель и считайте показания амперметра и вольтметра.
Рассчитать внутреннее сопротивление источника.
Определение ЭДС и внутреннего сопротивления источника тока графическим методом
Цель: для изучения измерений ЭДС, внутреннего сопротивления и тока короткого замыкания источника тока, основанных на анализе графика зависимости напряжения на выходе источника от силы тока в цепи.
Оборудование: гальванический элемент, амперметр, вольтметр, резистор R 1 , переменный резистор, ключ, зажимы, металлическая пластина, соединительные провода.
Из закона Ома для полной цепи следует, что напряжение на выходе источника тока зависит прямо пропорционально силе тока в цепи:
так как I = E/(R+r), то IR+ Ir = E, но IR = U, откуда U + Ir = E или U = E — Ir (1).
Если построить график зависимости U от I, то по точкам его пересечения с осями координат можно определить Е, I К.З. — сила тока короткого замыкания (ток, который потечет в цепи источника, когда внешнее сопротивление R станет равным нулю).
ЭДС определяется точкой пересечения графика с осью напряжений. Эта точка графика соответствует состоянию цепи, при котором ток в ней отсутствует и, следовательно, U = E.
Сила тока короткого замыкания определяется точкой пересечения графика с током ось. При этом внешнее сопротивление R = 0 и, следовательно, напряжение на выходе источника U = 0.
Внутреннее сопротивление источника находится по тангенсу наклона графика относительно оси тока . (Сравните формулу (1) с математической функцией вида Y = AX + B и запомните значение коэффициента при X).
Рабочий процесс
- После проверки схемы учителем соберите электрическую схему. Установите ползунок переменного резистора в положение, при котором сопротивление цепи, подключенной к источнику тока, будет максимальным.
Для записи результатов измерений подготовьте таблицу:
Определить значение тока в цепи и напряжение на клеммах источника при максимальном значении сопротивления переменного резистора. Внесите данные измерений в таблицу.
Повторить несколько раз измерение тока и напряжения, каждый раз уменьшая значение переменного сопротивления так, чтобы напряжение на клеммах источника уменьшилось на 0,1В. Прекратите измерение, когда ток в цепи достигнет 1А.
Нанесите точки, полученные в ходе эксперимента, на график. Постройте напряжение по вертикальной оси и ток по горизонтальной оси. Проведите прямую линию через точки.
Продолжить график до пересечения с осями координат и определить значения Е и, I К.З.
Измерьте ЭДС источника, подключив вольтметр к его клеммам при разомкнутой внешней цепи. Сравните значения ЭДС, полученные двумя методами, и укажите причину возможного расхождения результатов.
Определите внутреннее сопротивление источника тока. Для этого вычисляют тангенс наклона построенного графика к текущей оси. Так как тангенс угла в прямоугольном треугольнике равен отношению противолежащего катета к прилежащему, то это практически можно сделать, найдя отношение Е/I К.З
Лабораторная работа
«Измерение ЭДС и внутреннего сопротивления источника тока»
Дисциплина Физика
Виноградов А.Б.
Нижний Новгород
2014
Цель: формировать умение определять ЭДС и внутреннее сопротивление источника тока с помощью амперметра и вольтметра.
Оборудование: Выпрямитель ВУ-4М, амперметр, вольтметр, соединительные провода, табличка №1 элементы: ключ, резистор R 1 .
теоретический Содержание работы .
Внутреннее сопротивление источника тока.
При прохождении тока через замкнутой цепи электрически заряженные частицы движутся не только внутри проводников, соединяющих полюса источника тока, но и внутри самого источника тока. Поэтому в замкнутой электрической цепи различают наружный и внутренний участки цепи. Внешний участок цепи представляет собой всю совокупность проводников, которая подключается к полюсам источника тока. Внутренняя часть цепи является источником питания. Источник тока, как и любой другой проводник, имеет сопротивление. Так, в электрической цепи, состоящей из источника тока и проводников с электрическим сопротивлением R , электрический ток действует не только на внешний, но и на внутренний участок цепи. Например, при подключении к гальванической батарее карманного фонарика лампы накаливания электрическим током нагреваются не только катушка лампы и токоподводы, но и сама батарея. Электрическое сопротивление источника тока называется внутреннее сопротивление. В электромагнитном генераторе внутренним сопротивлением является электрическое сопротивление провода обмотки генератора. На внутреннем участке электрической цепи выделяется количество теплоты, равное
, где r — внутреннее сопротивление источника тока.
Суммарное количество теплоты, выделяющееся при протекании постоянного тока в замкнутой цепи, наружный и внутренний участки которой имеют сопротивления соответственно, равные Р и r , равно
. (2)
Любую замкнутую цепь можно представить в виде двух последовательно соединенных резисторов с эквивалентными сопротивлениями. Р и р . Следовательно, сопротивление полной цепи равно сумме внешнего и внутреннего сопротивлений:
Ом. Т.к. при последовательном соединении ток на всех участках цепи одинаков, то и по внешнему и внутреннему участкам цепи протекает одинаковый ток. Тогда по закону Ома для участка цепи падение напряжения на ее внешнем и внутреннем участках будет соответственно равно:
и
(3)
Электродвижущая сила.
Суммарная работа сил электростатического поля при движении зарядов по замкнутой цепи постоянного тока равна нулю. Следовательно, вся работа электрического тока в замкнутой электрической цепи совершается за счет действия внешних сил, вызывающих разделение зарядов внутри источника и поддерживающих постоянное давление на выходе источника тока. Отношение к работе
выполненные внешними силами для перемещения заряда q по цепочке, значение этого заряда называется источником электродвижущей силы (ЭДС) :
, (4)
, где
— портативная зарядка.
ЭДС выражается в тех же единицах, что и напряжение или разность потенциалов, т.е. в вольтах:
.
Закон Ома для полной цепи.
Если в результате прохождения постоянного тока в замкнутой электрической цепи происходит только нагрев проводников, то по закону сохранения энергии полная работа электрического тока в замкнутой цепи равна работе внешних сил источника тока, равно количеству теплоты, выделившейся на внешнем и внутреннем участках цепи:
. (5)
Из выражений (2), (4) и (5) получаем:
. (6)
Как
, затем
, (7)
или
. (8)
Сила тока в электрической цепи прямо пропорциональна электродвижущей силе источника тока и обратно пропорциональна сумме электрических сопротивлений внешнего и внутреннего участков цепи. Выражение (8) называется законом Ома для полной цепи.
Таким образом, с точки зрения физики Закон Ома выражает закон сохранения энергии для замкнутой цепи постоянного тока.
Заказ на работу .
Подготовка к работе.
Перед вами на столах минилаборатория по электродинамике. Его внешний вид представлен в л. Р. №9 на рис. 2.
Слева миллиамперметр, выпрямитель ВУ-4М, вольтметр, амперметр. Табличка №1 закреплена справа (см. рис. 3 в файле листа №9). В задней части корпуса размещены цветные соединительные провода: красный провод используется для подключения ВУ-4М к «+» гнезду планшета; белый провод – для подключения ВУ-4М к гнезду «-»; желтые провода – для подключения измерительных приборов к элементам планшета; синий — для соединения между собой элементов планшета. Секция закрывается откидной платформой. В рабочем положении платформа расположена горизонтально и используется как рабочая поверхность при сборке экспериментальных установок в экспериментах.
2. Ход работы.
В процессе работы вы освоите методику измерения основных характеристик источника тока, используя закон Ома для полной цепи, который связывает силу тока I в цепи, ЭДС источника тока, его внутреннее сопротивление r и сопротивление внешней цепи r соотношение:
. (9)
1 способ.
С экспериментальной установкой hema, показанной на рис. 1.
Рис.1.
Внимательно изучите. При разомкнутом переключателе В источник замыкается на вольтметр, сопротивление которого много больше внутреннего сопротивления источника ( r Р ). В этом случае ток в цепи настолько мал, что величиной падения напряжения на внутреннем сопротивлении источника можно пренебречь
, а ЭДС источника с пренебрежимо малой погрешностью равна напряжению на его зажимах , что измеряется вольтметром, т. е.
. (10)
Таким образом, ЭДС источника определяется по показаниям вольтметра при открытом ключе.
Если переключатель В замкнут, вольтметр покажет падение напряжения на резисторе Ч :
. (11)
Тогда на основании равенств (9), (10) и (11) можно утверждать, что
(12)
Из формулы (12) видно, что для определения внутреннего сопротивления источника тока необходимо кроме его ЭДС знать силу тока в цепи и напряжение на резисторе R при ключ закрыт.
Ток в цепи можно измерить амперметром. Проволочный резистор изготовлен из нихромовой проволоки и имеет сопротивление 5 Ом.
Соберите схему по схеме, представленной на рисунке 3.
После того, как схема собрана, нужно поднять руку, вызвать преподавателя для проверки правильности сборки электрической цепи. И если цепочка собрана правильно, то приступаем к работе.
При открытом ключе В снимите показания вольтметра и занесите значение напряжения в таблицу 1. Затем закройте ключ В и снова снимите показания вольтметра, но уже и показания амперметра. Внесите значения напряжения и тока в таблицу 1.
Рассчитать внутреннее сопротивление источника тока.
Таблица 1.
, АТ
, НА
Я, А
, НА
р , Ом
2-ходовой.
Сначала соберите экспериментальную установку, показанную на рисунке 2.
Рис. 2.
Измерьте силу тока в цепи с помощью амперметра, результат запишите в тетрадь. Сопротивление резистора =5 Ом. Все данные заносятся в таблицу 2., Ом
Контрольные вопросы :
Наружная и внутренняя части цепи.
Какое сопротивление называют внутренним? Обозначение.
Что такое полное сопротивление?
Дайте определение электродвижущей силы (ЭДС). Обозначение. Единицы.
Сформулируйте закон Ома для полной цепи.
Если бы мы не знали номиналов сопротивлений проволочных резисторов, можно ли было бы использовать второй способ и что для этого нужно сделать (может, например, нужно включить в схему какое-то устройство )?
Уметь собирать электрические схемы, используемые в работе.
Литература
Кабардин О.Ф. Исх. Материалы: Учеб. Руководство для студентов. — 3-е изд. — М.: Просвещение, 1991. — С.: 150-151.
Учебник для учащихся. Физика / Сост. Т. Фещенко, В. Вожегова.–М.: Филологическое общество «СЛОВО», ООО «Фирма», «Издательство АСТ», Центр гуманитарных наук при факультете журналистики МГУ. М. В. Ломоносова, 1998. — С.: 124,500-501.
Самойленко П.И. Физика (для нетехнических специальностей): Учеб. для общеобразовательных средних учреждений. Профессорское образование / П. И. Самойленко, А. В. Сергеев.-2-е изд., Стер.-М.: Издательский центр «Академия», 2003-с.: 181-182.
Предположим, имеется простая электрическая замкнутая цепь, включающая источник тока, например генератор, гальванический элемент или батарею, и резистор с сопротивлением R. Поскольку ток в цепи нигде не прерывается, он также протекает внутри источник.
В такой ситуации можно сказать, что любой источник имеет некоторое внутреннее сопротивление, препятствующее прохождению тока. Это внутреннее сопротивление характеризует источник тока и обозначается буквой r. Для любой батареи внутренним сопротивлением является сопротивление раствора электролита и электродов, для генератора сопротивление обмоток статора и т. д.
Таким образом, источник тока характеризуется как величиной ЭДС, так и величиной собственное внутреннее сопротивление r — обе эти характеристики говорят о качестве источника.
Электростатические высоковольтные генераторы (такие как генератор Ван де Граафа или генератор Вимшерста), например, имеют огромную ЭДС, измеряемую миллионами вольт, а их внутреннее сопротивление измеряется сотнями мегаом, поэтому они не подходит для создания больших токов.
Гальванические элементы (типа батареи) — наоборот — имеют ЭДС порядка 1 вольта, хотя их внутреннее сопротивление порядка долей или максимум десять ом, и поэтому токи единицы и десятки ампер можно получить от гальванических элементов.
На этой диаграмме показан реальный источник с подключенной нагрузкой. Здесь указаны его внутреннее сопротивление, а также сопротивление нагрузки. Согласно, сила тока в этой цепи будет равна:
Так как участок внешней цепи однородный, то из закона Ома можно найти напряжение на нагрузке:
Выразив сопротивление нагрузки из первой уравнение, а подставив его значение во второе уравнение, получим зависимость напряжения на нагрузке от тока в замкнутой цепи:
В замкнутой цепи ЭДС равна сумме падений напряжения на элементах внешней цепи и на внутреннем сопротивлении самого источника. Зависимость напряжения нагрузки от тока нагрузки идеально линейна.
На графике это видно, но экспериментальные данные на реальном резисторе (крестики возле графика) всегда отличаются от идеальных:
Эксперименты и логика показывают, что при нулевом токе нагрузки напряжение на внешней цепи равно равна ЭДС источника, а при нулевом напряжении на нагрузке ток в цепи равен . Это свойство реальных цепей помогает экспериментально найти ЭДС и внутреннее сопротивление реальных источников.
Экспериментальное определение внутреннего сопротивления
Для экспериментального определения этих характеристик постройте график зависимости напряжения нагрузки от тока, затем экстраполируйте его на пересечение с осями.
В точке пересечения графика с осью напряжения находится значение ЭДС источника, а в точке пересечения с осью тока величина тока короткого замыкания. В итоге внутреннее сопротивление находится по формуле:
Полезная мощность, развиваемая источником, распределяется на нагрузку. График зависимости этой мощности от сопротивления нагрузки показан на рисунке. Эта кривая начинается от пересечения осей координат в нулевой точке, затем увеличивается до значения максимальной мощности, после чего спадает до нуля при сопротивлении нагрузки, равном бесконечности.
Для нахождения максимального сопротивления нагрузки, при котором теоретически развивается максимальная мощность для данного источника, берется производная формулы мощности по R и приравнивается к нулю. Максимальная мощность будет развиваться при сопротивлении внешней цепи, равном внутреннему сопротивлению источника: 9 Ом.0005
Это утверждение о максимальной мощности при R = r позволяет экспериментально найти внутреннее сопротивление источника, построив зависимость мощности, выделяемой на нагрузке, от величины сопротивления нагрузки. Найдя реальное, а не теоретическое сопротивление нагрузки, обеспечивающее максимальную мощность, определяют реальное внутреннее сопротивление источника питания.
КПД источника тока показывает отношение максимальной мощности, выделяемой в нагрузку, к общей мощности, которая вырабатывается в данный момент
Калькулятор внутреннего сопротивления
Создано Purnima Singh, PhD
Отредактировано Wojciech Sas, PhD кандидатом и Стивеном Вудингом
Последнее обновление: 10 августа 2022 г.
Калькулятор внутреннего сопротивления Omni позволяет вам рассчитать сопротивление источника напряжения протекающему через него току .
Знаете ли вы, что и в грузовиках, и в мотоциклах используются батареи с одинаковой электродвижущей силой (ЭДС = 12 В)? Однако аккумулятор грузовика может дать гораздо больший заряд, чем аккумулятор мотоцикла, из-за меньшего внутреннего сопротивления.
Чтобы узнать больше о внутреннем сопротивлении, продолжайте читать эту статью. Вы узнаете:
- Что такое внутреннее сопротивление?
- Как найти внутреннее сопротивление батареи.
- Каково внутреннее сопротивление идеального источника напряжения/тока?
Что такое внутреннее сопротивление – определение внутреннего сопротивления
Внутреннее сопротивление источника напряжения (например, батареи) представляет собой сопротивление электролитов и электродов батареи протеканию тока через источник .
Внутреннее сопротивление новой батареи обычно низкое; однако по мере того, как батарея используется все чаще и чаще, ее внутреннее сопротивление увеличивается.
В следующем разделе мы узнаем, как найти внутреннее сопротивление батареи.
Как найти внутреннее сопротивление батареи – формула внутреннего сопротивления
Рассмотрим ячейку с электродвижущей силой (ЭДС) ε\varepsilonε и внутренним сопротивлением rrr.
Рисунок 1: Источник напряжения ЭДС ε и внутреннего сопротивления r, подключенный к внешнему резистору R.Если мы подключим эту ячейку к внешнему нагрузочному резистору RRR (см. рисунок 1), общее сопротивление (RTR_TRT) цепи станет :
RT=r+R\размер сноски R_T = r + RRT=r+R
И по формуле закона Ома ток III через цепь будет:
I=εR+r\footnotesize I = \frac{\varepsilon}{R+r}I=R+rε
Преобразовав приведенное выше уравнение, мы получим уравнение для внутреннего сопротивления:
r=εI−R\footnotesize \начать{выравнивать*} г = \ гидроразрыва {\ varepsilon} {I} — R \\ \конец{выравнивание*} r=Iε−R
Если мы знаем ток в цепи, мы также можем рассчитать напряжение на клеммах VVV батареи.
V=ε−Ir\размер сноски V = \varepsilon — IrV=ε−Ir
Напряжение на клемме батареи представляет собой выходное напряжение, измеренное на ее клемме . Приведенное выше уравнение показывает, что чем больше ток, тем ниже напряжение на клеммах. Точно так же, чем меньше внутреннее сопротивление, тем больше напряжение на клеммах.
Расчет внутреннего сопротивления – пример
В качестве примера рассчитаем внутреннее сопротивление элемента ЭДС 3 В3~\rm В3 В. Пусть ток в цепи равен 3 мА3\ \rm {мА}3 мА при сопротивлении нагрузки 995 Ом995\\Омега995 Ом подключено через него.
Заданы сопротивление нагрузки R=995 Ом R = 995\ \OmegaR=995 Ом, ЭДС ячейки ε=3 В\varepsilon = 3~\rm Vε=3 В, ток в цепи I=3 mAI = 3\ \rm {мА}I=3 мА.
Подставив эти значения в формулу внутреннего сопротивления, получим:
r=εI−R=3 V3×10−3 A−995 Ω=1000 Ω−995 Ω=5 Ω\quad \размер сноски \четверка \начать{выравнивать*} r &= \frac{\varepsilon}{I} — R\\[1em] & = \frac{3~\rm V}{3 \times 10^{-3}\ \rm {A}} — 9{-3}\ \rm {A} \times 5 \\Omega) \\[. 5em] & = 2,985 ~ \rm В \end{align*}V=ε−ir=3 V−(3×10−3 A×5 Ω)=2,985 В
Как использовать калькулятор внутреннего сопротивления
Теперь давайте посмотрим, как мы можем решить та же проблема с использованием нашего калькулятора внутреннего сопротивления:
Введите значение сопротивления нагрузки R=995 ΩR = 995\ \OmegaR=995 Ω.
Введите ЭДС ячейки (ε=3 В\varepsilon = 3~\rm Vε=3 В) и ток по цепи (I=3 мАв = 3\ \rm {мА}I=3 мА) в соответствующих полях. Вы можете выбрать раскрывающееся меню, чтобы выбрать соответствующую единицу измерения тока.
Калькулятор отобразит внутреннее сопротивление элемента (r=5 Ωr = 5 \\Omegar=5 Ω) и напряжение на клеммах (V=2,985 VV = 2,985 ~ \rm VV=2,985 V).
Если вас интересует расчет срока службы батареи на основе ее емкости и энергопотребления устройства, рекомендуем воспользоваться нашим калькулятором срока службы батареи.
Часто задаваемые вопросы
Как рассчитать внутреннее сопротивление батареи?
Чтобы рассчитать внутреннее сопротивление батареи, следуйте приведенным инструкциям:
Узнать ток по схеме.
Разделите ЭДС батареи на ток через цепь.
Вычесть сопротивление нагрузки из значения, полученного из шаг 2 .
Вы получите внутреннее сопротивление батареи.
Как рассчитать напряжение на клеммах аккумулятора?
Для расчета напряжения на клеммах выполните следующие действия:
Умножьте ток через цепь на внутреннее сопротивление батареи .
Вычесть значение из шаг 1 из ЭДС аккумулятора.
Поздравляем! Вы рассчитали напряжение на клеммах.
Каково внутреннее сопротивление идеального источника напряжения?
Ноль . Согласно закону Ома сопротивление определяется как r = V / I. Напряжение на идеальном источнике напряжения постоянно и не зависит от протекающего тока. Это означает, что кривая V-I будет прямой линией с нулевым наклоном . Отсюда можно сделать вывод, что внутреннее сопротивление идеального источника тока равно нулю.
Каково внутреннее сопротивление идеального источника тока?
Бесконечный . Ток, генерируемый идеальным источником тока, не зависит от приложенного к нему напряжения или сопротивления нагрузки. Если внутреннее сопротивление источника тока бесконечно, любое изменение сопротивления нагрузки не повлияет на выходной ток источника.
Пурнима Сингх, доктор философии
ЭДС ячейки (ε)
Сопротивление нагрузки (R)
Ток (I)
Внутреннее сопротивление (r)
Напряжение на клеммах (В)
Ознакомиться с 83 подобными калькуляторами электромагнетизма 🧲
Ускорение частицы в электрическом полеAC ваттРазмер выключателя… еще 80
10.
4 Оценка внутреннего сопротивления в цепях | Электрические цепиПредыдущий 10.3 Батареи и внутреннее сопротивление | Следующий 10.5 Расширение: мост Уитстона [не подлежит исследованию] |
Учебник Упражнение 10.2
Объясните, что подразумевается под внутренним сопротивлением реальной батареи.
Настоящие батареи изготавливаются из материалов, сопротивление. Это означает, что настоящие батареи — это не просто источники разность потенциалов (напряжение), но обладают и внутренней сопротивление.
Таким образом, внутреннее сопротивление является мерой сопротивления материала, из которого сделан аккумулятор.
Объясните, почему существует разница между ЭДС и напряжением на клеммах батареи, если нагрузка (внешняя сопротивление в цепи) сравнима по величине с внутренним сопротивлением батареи
ЭДС батареи практически постоянна потому что это зависит только от химической реакции (которая превращает химическое энергии в электрическую) происходит внутри батареи. Следовательно, мы видим, что напряжение на клеммах аккумулятора равно зависит от тока, потребляемого нагрузкой. Чем выше ток, тем понизьте напряжение на клеммах, потому что ЭДС постоянна. По По тем же соображениям напряжение равно ЭДС только тогда, когда ток очень маленький.
Чему равно внутреннее сопротивление батареи, если ее ЭДС равна \(\text{6}\) \(\text{V}\) и потенциал разница между его клеммами составляет \(\text{5,8}\) \(\text{V}\) при токе \(\text{0,5}\) \(\text{A}\) течет в цепи при подключении к нагрузке?
Падение напряжения на внутреннем сопротивлении равно разнице между ЭДС и напряжение на клеммах, когда батарея подключена к нагрузке. Внутреннее сопротивление мы рассматриваем как омический резистор последовательно с идеальной батареей. Мы знаем напряжение с и без нагрузки, и мы знаем ток, поэтому мы можем использовать закон Ома для определения внутреннее сопротивление:
\начать{выравнивать*} V_r&= Ir \\ (6)-(5,8)&= (0,5)r \\ г &= \фракция{0,2}{0,5} \\ & = \text{0,4}\text{ Ом} \конец{выравнивание*} \(\текст{0,4}\) \(\текст{Ω}\)
Батарея \(\text{12,0}\) \(\text{V}\) имеет внутреннее сопротивление \(\text{7,0}\) \(\text{Ом}\).
- Какой максимальный ток может обеспечить эта батарея?
- Какова разность потенциалов на его клеммах при подаче тока 150,0 мА?
- Нарисуйте схематический график, показывающий, как разность потенциалов на клеммах изменяется в зависимости от подаваемого тока, если внутреннее сопротивление остается постоянным. Как можно получить внутреннее сопротивление из графика?
Какой максимальный ток может обеспечить эта батарея?
\начать{выравнивать*} I _ {\ text {max}} & = \ frac {\ mathcal {E}} {r} \\ & =\ гидроразрыва {\ текст {12,0}} {\ текст {7,0}} \\ & =\текст{1,71428} \\ & =\текст{1,71}\текст{А} \конец{выравнивание*} 9{-\текст{1}})(\текст{7,0}) \\ & =\текст{10,95} \\ & =\текст{10,95}\текст{В} \конец{выравнивание*}
Нарисуйте схематический график, показывающий, как разность потенциалов на клеммах изменяется в зависимости от подаваемого тока, если внутреннее сопротивление остается постоянным. Как можно получить внутреннее сопротивление из графика?
Наклон графика представляет собой внутреннее сопротивление.
В слуховом аппарате батарея обеспечивает ток 25,0 мА через сопротивление 400 Ом. Когда объем увеличивается, сопротивление изменяется на 100 Ом (\Омега\) и ток возрастает до 60 мА. Что это ЭДС и внутреннее сопротивление ячейки? 9{-\text{2}}\) \(\text{Ω}\).
Батарея соединена последовательно с реостатом и амперметром. При сопротивлении резистора 10 \(\Омега\) сила тока 2,0 А. При сопротивлении 5 \(\Омега\) сила тока 3,8 А. Найти ЭДС и внутреннее сопротивление батареи.
У нас есть два неизвестных, но информация для двух разных сценариев, так что мы можем решать одновременно.
В первом случае:
\начать{выравнивать*} V & =\mathcal{E} — Ir \\ (\text{2})(\text{10}) = & =\mathcal{E} — (\text{2})r \\ \mathcal{E} & = (\text{2})(\text{10}) + (\text{2})r \конец{выравнивание*}
Во втором случае получим:
\начать{выравнивать*} V & =\mathcal{E} — Ir \\ (\text{3,8})(\text{5}) = & =\mathcal{E} — (\text{3,8})r \\ \mathcal{E} & = (\text{3,8})(\text{5}) + (\text{3,8})r \конец{выравнивание*}
Вычитание уравнения второго случая из уравнения первого случая:
\начать{выравнивать*} \mathcal{E} — \mathcal{E} & = ((\text{2})(\text{10}) + (\text{2})r) — ((\text{3,8})( \текст{5}) + (\текст{3,8})г)\\ 0 &= 20 + \text{2}r -19- \текст{3,8}р \\ r &= \frac{1}{\text{1,8}} \\ &=\текст{0,55555} \конец{выравнивание*}
Подставляем обратно в одно из уравнений, чтобы получить \(\mathcal{E}\):
\начать{выравнивать*} \mathcal{E} & = (\text{2})(\text{10}) + (\text{2})r \\ \mathcal{E} & = (\text{2})(\text{10}) + (\text{2})(\text{0,55555}) \\ \mathcal{E} & = \text{21,11111} \конец{выравнивание*}
ЭДС равна \(\text{21,11}\) \(\text{V}\), а внутреннее сопротивление равно \(\text{0,55}\) \(\text{Ом}\) .
Когда ячейка подключена непосредственно к высокоомному вольтметру, показания \(\текст{1,5}\) \(\текст{V}\). Когда ячейка закорочена через амперметр низкого сопротивления, ток равен \(\текст{2,5}\) \(\текст{А}\). Чему равны ЭДС и внутреннее сопротивление элемента?
В случае вольтметра с очень высоким сопротивлением мы должны предположить, что ток равен нулю. Это означает что измерение на самом деле представляет собой ЭДС, а разность потенциалов на внутреннем сопротивлении подчиняется закону Ома. закон, \(V=Ir=(0)r=0\).
В случае амперметра с очень низким сопротивлением мы можем предположить, что текущий ток является максимальным ток возможен. Мы знаем ЭДС и максимальный ток, поэтому можем найти \(r\):
\начать{выравнивать*} {I}_{c}& = \frac{\mathcal{E}}{r} \\ (\text{2,5})&= \frac{\text{1,5}}{r} \\ r&= \frac{\text{1,5}}{\text{2,5}} \\ г & = \текст{0,6}\текст{ Ом} \конец{выравнивание*}
ЭДС равна \(\text{1,5}\) \(\text{V}\), а внутреннее сопротивление равно \(\text{0,6}\) \(\text{Ω}\) .
10.4 Оценка внутреннего сопротивления в цепях (ESCPW)
Подход (ESCPX)
Подход к решению проблем, связанных с внутренним сопротивлением батарей, прост, вы просто нужно понимать, что каждая батарея в предыдущих примерах была источником ЭДС, \(\mathcal{E}\), и небольшой резистор, \(r\), а затем решите как раньше, но включите \(r\) в свои расчеты.
Важно понимать, что разность потенциалов, которую вы рассчитали или Примеры — это не ЭДС, это ЭДС за вычетом разности потенциалов на внутреннем сопротивлении.
Чтобы подчеркнуть, что внутреннее сопротивление является продолжением того, что вы уже сделали, мы возьмем предыдущие рабочие примеры и рассмотрим внутреннее сопротивление батареи. Если внутреннее сопротивление не вести себя как омический резистор, это было бы невозможно, но мы не будем иметь дело с такими случаями.
Приложения (ESCPY)
Рабочий пример 7: Внутреннее сопротивление в цепи с последовательно включенными резисторами
Для следующей цепи рассчитайте:
разность потенциалов \(V_\text{1}\), \(V_\text{2}\) и \(V_\text{3}\) на резисторах \(R_\text{1}\), \(R_\text{2}\) и \(R_\text{3}\)
.сопротивление \(R_\text{3}\).
сопротивление \(R_\text{3}\).
Если внутреннее сопротивление равно \(\text{0,1}\) \(\text{Ом}\), какова ЭДС батареи и какая мощность рассеивается на внутреннем сопротивлении батареи?
Примечание
Это очень похоже на вопрос, который вы видели ранее. Это для того, чтобы выделить тот факт, что подход при работе с внутренним сопротивлением строится на все том же принципы, с которыми вы уже работали.
Определите, как подойти к задаче
Нам даны разность потенциалов на ячейке и ток в цепи, а также сопротивления двух из трех резисторов. Мы можем использовать закон Ома, чтобы вычислить разность потенциалов на известные резисторы. Поскольку резисторы включены последовательно, разность потенциалов равна \(V = V_\text{1} + V_\text{2} + V_\text{3}\), и мы можем вычислить \(V_\text{3}\). Теперь мы можем использовать эту информацию найти разность потенциалов на неизвестном резисторе \(R_\text{3}\).
Рассчитать разность потенциалов на \(R_\text{1}\)
Используя закон Ома: \начать{выравнивать*} R_\text{1} &= \frac{V_\text{1}}{I} \\ I \cdot R_\text{1} &= I \cdot \frac{V_\text{1}}{I} \\ V_\text{1} &= {I}\cdot{R_\text{1}}\\ &= 2 \cdot 1 \\ В_\текст{1} &= \текст{2}\текст{В} \конец{выравнивание*}
Рассчитайте разность потенциалов на \(R_\text{2}\)
Опять же, используя закон Ома: \начать{выравнивать*} R_\text{2} &= \frac{V_\text{2}}{I} \\ I \cdot R_\text{2} &= I \cdot \frac{V_\text{2}}{I} \\ V_\text{2} &= {I}\cdot{R_\text{2}}\\ &= 2 \cdot 3 \\ В_\текст{2} &= \текст{6}\текст{В} \end{выравнивание*}
Рассчитать разность потенциалов на \(R_\text{3}\)
Поскольку разность потенциалов на всех объединенных резисторах должна быть такой же, как потенциал разность между ячейками в последовательной цепи, мы можем найти \(V_\text{3}\), используя: \начать{выравнивать*} В &= В_\текст{1} + В_\текст{2} + В_\текст{3}\\ В_\текст{3} &= В — В_\текст{1} — В_\текст{2} \\ &= 23 — 2 — 6 \\ В_\текст{3}&= \текст{15}\текст{В} \end{выравнивание*}
Найдите сопротивление \(R_\text{3}\)
Мы знаем разность потенциалов на \(R_\text{3}\) и ток через него, поэтому мы можем использовать закон Ома для расчета значения сопротивления: \начать{выравнивать*} R_\text{3} &= \frac{V_\text{3}}{I}\\ &= \frac{\text{15}}{\text{2}} \\ R_\text{3}&= \text{7,5}~\Omega \конец{выравнивание*}
Разность потенциалов внутреннего сопротивления батареи
Значение ЭДС можно рассчитать по разности потенциалов нагрузки и потенциала разница внутреннего сопротивления. 2}{R}\), и мы знаем ток в цепи, внутреннее сопротивление и разность потенциалов на ней, поэтому мы можем использовать любые форма уравнения для мощности:
\начать{выравнивать*} P_r &= V_rI_r\\ & = (\текст{0,2})(\текст{2})\\ & = \текст{0,4}\текст{W} \конец{выравнивание*}
Напишите окончательный ответ
- \(V_\text{1} = \text{2,0}\text{ V}\)
- \(В_\текст{2} = \текст{6,0}\текст{В}\)
- \(V_\text{3} = \text{10,0}\text{V}\)
- \(R_\text{3} = \text{7,5} \Omega\)
- \(\mathcal{E} = \text{23,2}\text{V}\)
- \(P_r = \text{0,4}\text{W}\)
Рабочий пример 8: Внутреннее сопротивление и параллельные резисторы
Разность потенциалов на батарее составляет 18 В, когда она подключена к двум параллельным резисторам \(\text{4,00}\) \(\Omega\) и \(\text{12,00}\) \(\Omega\) соответственно. Рассчитать ток через ячейки и через каждый из резисторов. Если внутреннее сопротивление батареи равно \(\text{0,375}\) \(\text{Ω}\) чему равна ЭДС батареи?
Сначала нарисуйте схему, прежде чем делать какие-либо расчеты
Определите, как подойти к проблеме
Нам нужно определить ток через ячейку и каждый из параллельных резисторов. Нам дали разность потенциалов на ячейке и сопротивление резисторов, поэтому мы можем использовать закон Ома для рассчитать ток.
Расчет тока через элемент
Для расчета тока через элемент нам сначала нужно определить эквивалентное сопротивление остальных цепи. Резисторы включены параллельно, поэтому: \начать{выравнивать*} \frac{\text{1}}{R} &= \frac{\text{1}}{R_\text{1}} + \frac{\text{1}}{R_\text{2}} \ \ &= \frac{\text{1}}{\text{4}} + \frac{\text{1}}{\text{12}} \\ &= \frac{3+1}{\text{12}} \\ &= \frac{\text{4}}{\text{12}} \\ R &= \frac{\text{12}}{\text{4}} = \text{3,00} \\Omega \конец{выравнивание*} Теперь, используя закон Ома, найдем ток через ячейку: \начать{выравнивать*} R &= \frac{V}{I} \\ I &= \frac{V}{R} \\ &= \frac{\text{18}}{\text{3}} \\ Я &= \text{6,00}\text{A} \end{выравнивание*}
Теперь определите ток через один из параллельных резисторов
Мы знаем, что для чисто параллельной конфигурации резисторов разность потенциалов на ячейке равна такая же, как разность потенциалов на каждом из параллельных резисторов. Для этой схемы: \начать{выравнивать*} В &= В_\текст{1} = В_\текст{2} = \текст{18}\текст{В} \конец{выравнивание*} Начнем с расчета тока через \(R_\text{1}\) по закону Ома: \начать{выравнивать*} R_\text{1} &= \frac{V_\text{1}}{I_\text{1}} \\ I_\text{1} &= \frac{V_\text{1}}{R_\text{1}} \\ &= \frac{\text{18}}{\text{4}} \\ I_\text{1} &= \text{4,50}\text{A} \end{выравнивание*}
Рассчитайте ток через другой параллельный резистор
Мы можем снова использовать закон Ома, чтобы найти ток в \(R_\text{2}\): \начать{выравнивать*} R_\text{2} &= \frac{V_\text{2}}{I_\text{2}} \\ I_\text{2} &= \frac{V_\text{2}}{R_\text{2}} \\ &= \frac{\text{18}}{\text{12}} \\ I_\text{2} &= \text{1,50}\text{A} \конец{выравнивание*} Альтернативным методом расчета \(I_\text{2}\) было бы использование того факта, что токи через каждый из параллельных резисторов должен в сумме составлять полный ток через ячейку: \начать{выравнивать*} I &= I_\text{1} + I_\text{2} \\ I_\text{2} &= I — I_\text{1} \\ &= 6 — 4,5 \\ I_\text{2} &= \text{1,5}\text{A} \end{выравнивание*}
Определить ЭДС
Суммарный ток через батарею представляет собой ток через внутреннее сопротивление батареи. Знание тока и сопротивления позволяет нам использовать закон Ома для определения разности потенциалов на внутреннее сопротивление и, следовательно, ЭДС батареи.
Используя закон Ома, мы можем определить разность потенциалов на внутреннем сопротивлении:
\начать{выравнивать*} V &=I \cdot r \\ &=\текст{6} \cdot \текст{0,375} \\ &= \текст{2,25}\текст{В} \конец{выравнивание*}
Мы знаем, что ЭДС батареи представляет собой разность потенциалов на выводе, суммированную с разность потенциалов на внутреннем сопротивлении так:
\начать{выравнивать*} \mathcal{E}& = V+Ir \\ & = \текст{18}+\текст{2,25} \\ & = \текст{20,25}\текст{В} \конец{выравнивание*}
Напишите окончательный ответ
Ток через ячейку равен \(\text{6,00}\) \(\text{A}\).
Ток через резистор \(\text{4,00}\) \(\Omega\) равен \(\text{4,50}\) \(\text{A}\).
Ток через резистор \(\text{12,00}\) \(\Omega\) равен \(\text{1,50}\) \(\text{A}\).
ЭДС батареи равна \(\text{20,25}\) \(\text{V}\).
Рабочий пример 9: Мощность в последовательной и параллельной сети резисторов
Для следующей цепи:
Ток, выходящий из батареи, равен \(\text{1,07}\) \(\text{A}\), общая мощность, рассеиваемая во внешнем цепь \(\text{6,42}\) \(\text{W}\), отношение полных сопротивлений двух параллельных цепей \(R_{P\text{1}} : R_{P\text{2}}\) равно 1:2, отношение \(R_\text{1} : R_\text{2}\) равно 3: 5 и \(R_\text{3}=\text{7,00}\text{Ω}\).
Определить:
- разность потенциалов аккумулятора,
- мощность, рассеиваемая в \(R_{P\text{1}}\) и \(R_{P\text{2}}\), а
- , если батарея помечена как имеющая ЭДС \(\text{6,50}\) \(\text{V}\), каково значение сопротивление каждого резистора и мощность, рассеиваемая в каждом из них.
Что требуется
В этом вопросе вам дается различная информация и предлагается определить мощность, рассеиваемую в каждый резистор и каждая комбинация резисторов. Обратите внимание, что приведенная информация в основном относится к общему схема. Это ключ к тому, что вы должны начать с общей схемы и двигаться вниз к более конкретным элементы цепи.
Вычисление разности потенциалов батареи
Сначала мы сосредоточимся на батарее. Нам дана мощность для всей цепи, а также ток оставив батарею. Мы знаем, что разность потенциалов на клеммах батареи равна разность потенциалов по цепи в целом.
Мы можем использовать соотношение \(P=VI\) для всей цепи, потому что разность потенциалов такая же, как разность потенциалов на клеммах аккумулятора: \начать{выравнивать*} Р &=VI \\ V &= \frac{P}{I} \\ &=\frac{\text{6,42}}{\text{1,07}} \\ &= \текст{6,00}\текст{В} \end{выравнивание*}
Разность потенциалов на аккумуляторе равна \(\text{6,00}\) \(\text{V}\).
Мощность, рассеиваемая в \(R_{P\text{1}}\) и \(R_{P\text{2}}\)
Помните, что мы работаем от общей схемы к деталям для отдельных элементов, это противоположно тому, как вы обращались с этой схемой ранее.
Мы можем рассматривать параллельные сети как эквивалентные резисторы, поэтому схема, с которой мы сейчас имеем дело с выглядит так:
Мы знаем, что ток через два элемента цепи будет одинаковым, потому что это последовательная цепь. и что сопротивление всей цепи должно быть: \(R_{Ext}=R_{P\text{1}}+R_{P\text{2}}\). Мы можем определить полное сопротивление из закона Ома для цепи в целом: \начать{выравнивать*} V_{батарея}&=IR_{Внешний} \\ R_{Ext} &=\frac{V_{батарея}}{I} \\ &=\frac{\text{6,00}}{\text{1,07}}\\ &=\текст{5,61}\текст{Ом} \end{выравнивание*}
Мы знаем, что соотношение между \(R_{P\text{1}} : R_{P\text{2}}\) равно 1:2, что означает, что мы знаем: \начать{выравнивать*} R_{P\text{1}} &= \frac{\text{1}}{\text{2}}R_{P\text{2}} \ \ \text{and} \\ R_T &= R_{P\текст{1}} + R_{P\текст{2}} \\ & = \frac{\text{1}}{\text{2}}R_{P\text{2}} + R_{P\text{2}} \\ &=\frac{\text{3}}{\text{2}}R_{P\text{2}} \\ (\text{5,61}) &=\frac{\text{3}}{\text{2}}R_{P\text{2}} \\ R_{P\text{2}} &= \frac{\text{2}}{\text{3}}(\text{5,61}) \\ R_{P\text{2}} &= \text{3,74}\text{ Ом} \конец{выравнивание*} и поэтому: \начать{выравнивать*} R_{P\text{1}} &= \frac{\text{1}}{\text{2}}R_{P\text{2}} \\ &=\frac{\text{1}}{\text{2}}(3. 74) \\ &= \text{1,87}\text{ Ом} \end{выравнивание*} 92(\текст{3,74}) \\ &= \текст{4,28}\текст{W} \конец{выравнивание*} Эти значения добавятся к исходному значению мощности, которое мы имели для внешней цепи. если бы они не мы бы допустили ошибку в расчетах.
Расчеты параллельной сети 1
Теперь мы можем приступить к детальным расчетам для первого набора параллельных резисторов.
Мы знаем, что отношение между \(R_{\text{1}} : R_{\text{2}}\) равно 3:5, что означает, что мы знаем \(R _ {\ text {1}} = \ frac {\ text {3}} {\ text {5}} R _ {\ text {2}} \). Мы также знаем общее сопротивление двух параллельных резисторов в этой сети. это \(\text{1,87}\) \(\text{Ω}\). Мы можем использовать соотношение между значениями двух резисторов, а также формула для общего сопротивление (\(\frac{\text{1}}{R_PT}=\frac{\text{1}}{R_\text{1}}+\frac{\text{1}}{R_\text{2 }}\)) чтобы найти номиналы резисторов: \начать{выравнивать*} \frac{\text{1}}{R_{P\text{1}}}&=\frac{\text{1}}{R_\text{1}}+\frac{\text{1}}{ Р_\текст{2}} \\ \frac{\text{1}}{R_{P\text{1}}}&=\frac{\text{5}}{3R_\text{2}}+\frac{\text{1}}{ Р_\текст{2}} \\ \frac{\text{1}}{R_{P\text{1}}}&=\frac{\text{1}}{R_\text{2}}(\frac{\text{5}}{ \text{3}}+1) \\ \frac{\text{1}}{R_{P\text{1}}}&=\frac{\text{1}}{R_\text{2}}(\frac{\text{5}}{ \text{3}}+\frac{\text{3}}{\text{3}}) \\ \ frac {\ text {1}} {R_ {P \ text {1}}} & = \ frac {\ text {1}} {R_ \ text {2}} \ frac {\ text {8}}} {\ текст{3}} \\ R_\text{2}&=R_{P\text{1}}\frac{\text{8}}{\text{3}} \\ &=(\text{1,87})\frac{\text{8}}{\text{3}} \\ &=\текст{4,99}\text{ Ом} \конец{выравнивание*} Мы также можем вычислить \(R_{\text{1}}\): \начать{выравнивать*} R _ {\ text {1}} & = \ frac {\ text {3}} {\ text {5}} R _ {\ text {2}} \\ &= \frac{\text{3}}{\text{5}}(\text{4,99}) \\ &= \text{2,99}\text{ Ом} \end{align*}
Для определения мощности нам понадобится рассчитанное нами сопротивление и либо разность потенциалов или текущий. Два резистора включены параллельно, поэтому разность потенциалов на них одинакова. такая же, как разность потенциалов в параллельной сети. Мы можем использовать закон Ома, чтобы определить разность потенциалов в сети параллельных резисторов, так как мы знаем общее сопротивление и знаем электрический ток: \начать{выравнивать*} V &= I R \\ &=(\текст{1,07})(\текст{1,87}) \\ &=\текст{2,00}\текст{В} \end{выравнивание*} 92}{\текст{4,99}} \\ &=\текст{0,80}\текст{W} \конец{выравнивание*}
Расчет параллельной сети 2
Теперь мы можем приступить к детальному расчету второго набора параллельных резисторов.
Нам дано \(R_\text{3}=\text{7,00}\text{ Ω}\), и мы знаем \(R_{P\text{2}}\), поэтому мы можем вычислить \(R_\text{4}\) из: \начать{выравнивать*} \frac{\text{1}}{R_{P\text{2}}} &= \frac{\text{1}}{R_\text{3}}+\frac{\text{1}}{ Р_\текст{4}} \\ \frac{\text{1}}{\text{3,74}} &= \frac{\text{1}}{\text{7,00}}+\frac{\text{1}}{R_ \текст{4}} \\ R_\text{4}&=\text{8,03}\text{ Ом} \end{выравнивание*} 92}{\текст{8,03}} \\ &=\текст{1,99}\текст{W} \конец{выравнивание*}
Внутреннее сопротивление
Мы знаем, что ЭДС батареи равна \(\text{6,5}\) \(\text{V}\), но что разность потенциалов измерено через терминалы только \(\text{6}\) \(\text{V}\). Разница есть разность потенциалов через внутреннее сопротивление батареи, и мы можем использовать известный ток и закон Ома, чтобы определить внутреннее сопротивление:
\начать{выравнивать*} V&=I \cdot R \\ R&=\frac{V}{I} \\ & = \ гидроразрыва {\ текст {0,5}} {\ текст {1,07}} \\ & = \текст{0,4672897}\\ & = \text{0,47}\text{ Ом} \конец{выравнивание*}
Мощность, рассеиваемая внутренним сопротивлением батареи:
\начать{выравнивать*} Р &=VI \\ & = \текст{0,5}\cdot\текст{1,07} \\ &=\текст{0,535}\текст{W} \конец{выравнивание*}
Рабочий пример 10: Внутреннее сопротивление и фары [NSC 2011 Paper 1]
Фара и два ОДИНАКОВЫХ задних фонаря скутера подключены параллельно к аккумулятору с неизвестным внутреннее сопротивление, как показано на упрощенной принципиальной схеме ниже. Фара имеет сопротивление \(\text{2,4}\) \(\text{Ω}\) и управляется переключателем \(\textbf{S}_1\). Задние фонари управляются переключателем \(\textbf{S}_2\). Сопротивлением соединительных проводов можно пренебречь.
На приведенном рядом графике показана разность потенциалов на клеммах аккумулятора до и после переключатель \(\textbf{S}_1\) закрыт (пока переключатель \(\textbf{S}_2\) открыт). Переключатель \(\textbf{S}_1\) есть закрыто во время \(\textbf{t}_1\).
Используйте график для определения ЭДС батареи.
(1 балл)
ПРИ ТОЛЬКО ВЫКЛЮЧАТЕЛЕ \(\textbf{S}_1\) ЗАМКНУТ, вычислите следующее:
Ток через фару
(3 балла)
Внутреннее сопротивление, Ом, батареи
(3 балла)
ОБА ПЕРЕКЛЮЧАТЕЛЯ \(\textbf{S}_1\) И \(\textbf{S}_2\) ЗАКРЫТЫ. Аккумулятор выдает ток \(\text{6}\) \(\text{A}\) в течение этого периода.
Рассчитайте сопротивление каждого заднего фонаря.
(5 баллов)
Как повлияет на показания вольтметра перегорание фары? (Оба переключателя \(\textbf{S}_1\) и \(\textbf{S}_2\) все еще закрыты.)
Запишите только УВЕЛИЧЕНИЕ, УМЕНЬШЕНИЕ или ОСТАЕТСЯ ИЗМЕНЕННЫМ.
Дайте объяснение.
(3 балла)
Вопрос 1
\(\текст{12}\) \(\текст{V}\)
(1 балл)
Вопрос 2.1
Опция 1:
\начать{выравнивать*} I & = \frac{V}{R} \\ & = \ гидроразрыва {\ текст {9,6}} {\ текст {2,4}} \\ & = \текст{4 А} \конец{выравнивание*}Вариант 2:
\начать{выравнивать*} \text{ЭДС} & = ИК + ИК \\ 12 & = I(\text{2,4}) + \text{2,4} \\ \поэтому я & = \text{4 A} \конец{выравнивание*}(3 балла)
Вопрос 2. 2
Опция 1:
\начать{выравнивать*} \text{ЭДС} & = ИК + Ir \\ 12 & = \text{9,4} + 4r \\ г & = \текст{0,6}\\Омега \конец{выравнивание*}Вариант 2:
\начать{выравнивать*} V_{потерянный} & = Ir \\ \text{2,4} & = \text{4}r \\ \поэтому r & = \text{0,6}\ \Omega \конец{выравнивание*}Вариант 3:
\начать{выравнивать*} \text{ЭДС} & = I(R + r) \\ \text{12} & = \text{4}(\text{2,4} + r)\\ \поэтому r & = \text{0,6}\ \Omega \конец{выравнивание*}(3 балла)
Вопрос 3
Опция 1:
\начать{выравнивать*} \text{ЭДС} & = ИК + Ir \\ \text{12} & = \text{6}(R + \text{0,6}) \\ R _ {\ text {ext}} & = \ text {1,4} \ \ Omega \end{выравнивание*}\begin{выравнивание*} \frac{1}{R} & = \frac{1}{R_{1}} + \frac{1}{R_{2}} \\ \frac{1}{\text{1,4}} & = \frac{1}{\text{2,4}} + \frac{1}{R} \\ R & = \text{3,36}\\Омега \конец{выравнивание*}Каждый задний фонарь: \(R = \text{1,68}\\Omega\)
Вариант 2:
\начать{выравнивать*} \text{ЭДС} & = V_{\text{терминал}} + Ir \\ 12 & = V _ {\ text {терминал}} + 6 (\ text {0,6}) \\ \поэтому V_{\text{терминал}} & = \text{8,4}\text{V} \end{выравнивание*}\begin{выравнивание*} I _ {\ text {2,4} \ \ Omega} & = \ frac {V} {R} \\ &= \frac{\text{8,4}}{\text{2,4}} \\ &= \текст{3,5 А} \end{выравнивание*}\begin{выравнивание*} I _ {\ text {задние фонари}} & = 6 — \ text {3,5} \\ & = \текст{2,5}\текст{А} \\ R _ {\ text {задние фонари}} & = \ frac {V} {I} \\ & = \ гидроразрыва {\ текст {8,4}} {\ текст {2,5}} \\ & = \текст{3,36}\\Омега\\ R _ {\ text {задний фонарь}} & = \ text {1,68} \ \ Omega \конец{выравнивание*}Вариант 3:
\начать{выравнивать*} V & = ИК \\ \text{12} & = \text{6}(R)\\ R _ {\ text {ext}} & = 2 \ \ Omega \end{выравнивание*}\begin{выравнивание*} R_{\text{параллельно}} & = 2 — \text{0,6} \\ & = \текст{1,4}\\Омега\\ \frac{1}{R} & = \frac{1}{R_{1}} + \frac{1}{R_{2}} \\ \frac{1}{\text{1,4}} & = \frac{1}{\text{2,4}} + \frac{1}{R} \\ R & = \text{3,36}\\Омега \конец{выравнивание*}Каждый задний фонарь: \(R = \text{1,68}\ \Omega\)
Вариант 4:
Для параллельной комбинации: \(I_{1} + I_{2} = 6\text{A}\)
\начать{выравнивать*} \поэтому \frac{V}{\text{2,4}} + \frac{V}{R_{\text{задние фонари}}} & = \text{6} \\ \text{8,4}\left(\frac{1}{\text{2,4}} + \frac{1}{R_{\text{задние фонари}}} \right) & = \text{6 }\\ \поэтому R _{\text{задние фонари}} & = \text{3,36}\ \Omega \\ R _ {\ text {задний фонарь}} & = \ text {1,68} \ \ Omega \конец{выравнивание*}(5 баллов)
Вопрос 4
Увеличивает
Сопротивление увеличивается, а ток уменьшается. Таким образом, \(Ir\) (потерянные вольты) должны уменьшаться, что приводит к увеличение напряжения.
(3 балла)
[ВСЕГО: 15 баллов]
временный текстПредыдущий 10.3 Батареи и внутреннее сопротивление | Оглавление | Следующий 10.5 Расширение: мост Уитстона [не подлежит исследованию] |
Измерение ЭДС и внутреннего сопротивления элемента
Все новые спецификации включают «измерение внутреннего сопротивления элемента» в качестве одного из практических методов. Это, вероятно, новая часть физики для ваших учеников, и, хотя практические настройки просты, сбор и обработка данных являются более сложной задачей. Сравнение двух разных типов клеток, как показано в этом фильме, может сделать практическое применение более интересным, с потенциалом для дифференциации по способностям.
Что в фильме
Фильм начинается (до 1:24) с теории, которую вы, вероятно, познакомите со студентами перед выполнением практических занятий.
Начиная с 1:30, фильм показывает, как можно проводить практические занятия с обычными элементами питания, а также с элементом-таблеткой (батарейка для часов).
Безопасность
Кристина и Алом делают несколько вещей в фильме, чтобы ограничить ток, чтобы ячейка не перегревалась: они используют ограничительный резистор, начинают с малых токов и замыкают цепь только на мгновение. Это представляет собой безопасную рабочую практику, но нагрев ячейки также повлияет на сопротивление, которое мы пытаемся измерить.
Ячейка AA
Мы использовали резистор 10 Ом для ограничения тока в цепи. Подойдет и простой постоянный резистор, но убедитесь, что он выдерживает максимальную мощность, которую вы ожидаете от схемы, — несколько ватт. У нас не было под рукой такого резистора для съемок, отсюда и огромный блок переключаемых сопротивлений.
Для изменения тока для получения нескольких показаний мы использовали старый реостат, рассчитанный примерно на 16 Ом. На практике все с диапазоном до 50 Ом или около того должно работать. Также можно использовать ряд различных постоянных резисторов или блок переключаемых сопротивлений.
Вместо мультиметров можно использовать цифровые или аналоговые вольтметры или амперметры, но, как указывает Кристина в фильме, использование мультиметров — это навык, который ваши ученики должны развивать в любом случае. Учащимся необходимо будет выбрать наиболее подходящий диапазон, который, вероятно, будет составлять 20 В постоянного тока для вольтметра и 200 мА постоянного тока для амперметра (следя за тем, чтобы преобразовать обратно в ампер при обработке данных).
Мультиметровые фильмы Алома могут быть скучными, но они набрали треть миллиона просмотров, так что… они могут иметь некоторые достоинства. Нажмите на YouTube или разверните фильм из этих крошечных окон:
Измерение напряжения с помощью мультиметра
Измерение тока с помощью мультиметра
Сбор и обработка данных
Работая в парах, этот эксперимент можно провести очень быстро. Систематические данные — это хорошо, но при наличии хорошего разброса точек данных по всему диапазону течений учащиеся должны получить хороший результат.
Из наших данных мы получили:
Градиент = -2,10
y-точка = 1,415
итак:
ЭДС = 1,415 В
Внутреннее сопротивление = 2,10 Ом
Обычно мы ожидаем, что элемент AA будет иметь ЭДС около 1,5 В и внутреннее сопротивление около 1 Ом. Наши были старыми и дешевыми, что, вероятно, объясняет наши результаты: стоит отметить, что клетки более низкого качества могут сделать эксперимент более интересным!
Вы можете спросить своих учеников:
- Является ли их результат тем, что они ожидают от упаковки или этикетки ячейки?
- Как они могли оценить неопределенность своих данных?
Кристина упоминает допуск на 4 фута 11 дюймов, что является концепцией, с которой учащиеся могут быть незнакомы. Все компоненты имеют установленный производителем допуск, в котором указан диапазон ±%, который можно ожидать при нормальном использовании компонента.
Батарея типа «таблетка»
Мы использовали стандартную батарейку для часов CR2032. Показания для такого типа ячеек различаются гораздо сильнее, чем для ячейки АА. Мы предполагали, что это связано с внутренним нагревом клетки, но при написании этих заметок мы начали задаваться вопросом, не связано ли это больше с химией, которая происходит внутри — если есть предел скорости реакции, это могло бы объяснить, почему напряжение быстро падает (особенно в случаях сильноточных стоков), прежде чем ячейка восстанавливается после «отдыха». Комментарии приветствуются, а пока мы продолжим…
Фотографирование счетчиков — один из способов справиться с быстро меняющимися показаниями. Другим подходом может быть использование аналоговых счетчиков, которые легче считывать на глаз.
На отметке 6:09 вы увидите Кристину, использующую «наиболее подходящую» линейку — прозрачную линейку с прорезью посередине. Мы рекомендуем это! Наши результаты:
В связи с возросшей неопределенностью показаний Алом предлагает повторить весь эксперимент дважды. Каждый повтор можно нанести на одни и те же оси и сравнить градиенты и точки пересечения по оси Y. Затем учащиеся могли найти среднее значение ЭДС и внутреннего сопротивления, а также связанные с ними погрешности.
Обычно мы ожидаем, что 3-вольтовая ячейка будет иметь ЭДС около 3 В, а внутреннее сопротивление намного выше, чем у ячейки AA, что мы и обнаружили, измерив внутреннее сопротивление 15 Ом.
Вы можете спросить своих учеников:
- Есть ли лучший способ записывать колебания показаний?
- Является ли простое среднее допустимым способом объединения повторных показаний?
- Как лучше всего поступить с данными, которые на графике выглядят сгруппированными из-за того, что вам нужно включить точку пересечения по оси Y? (Здесь вы можете изучить математические методы экстраполяции.)
- Почему ячейки имеют разные ЭДС и внутреннее сопротивление? Какие химические вещества они содержат и как они устроены внутри? (полезным ресурсом здесь является Battery University, хотя он становится немного… подробным, скажем так?)
Другие примечания
Стоимость
- 50 элементов AA должны стоить около 12 фунтов стерлингов.
- 40 Плоские батарейки на 3 В должны стоить около 5 фунтов стерлингов.
Дальнейшая работа
Некоторым учителям нравится ставить перед своими учениками дополнительные задачи, исследуя ЭДС и внутреннее сопротивление элемента, состоящего из медных и цинковых электродов и предмета из фруктов или овощей, например: «картофельной батареи». об этом можно найти на веб-сайте «Практическая физика». Нарезка картофеля на разные формы может сделать интересное сравнение.
Оценка
Общие практические критерии оценки
На момент написания, экзаменационные комиссии пришли к соглашению, что это практическое задание может быть использовано для решения, полностью или частично:
- CPAC 1: следует письменным процедурам
- Правильно следует инструкциям по проведению экспериментальных методик или процедур
- CPAC 4: Делает и записывает наблюдения.
- Делает точные наблюдения, относящиеся к экспериментальной или следственной процедуре.